quinta-feira, 5 de junho de 2014

" Se derrubarem essa se constrói outra. " E sobre as pessoas que possam vir a perder a vida em outro atentado ? 'Constrói' outras também ?‎

Mas não se pode proceder na vida com base na suposição de que o que se vai fazer seja objeto de atentados, se se está fazendo algo bom e não criminoso. Senão não se faz nada na vida, por temor de que seja roubado ou que se seja agredido por fazer justiça ou bondade. Não se pode deixar o crime prevalecer porque os criminosos atemorizam a população. Tem-se que enfrentar o crime com coragem. Caso contrário se capitulará a ele e se deixará que a Terra seja dominada pelo mal. Não estou dizendo que os Estados Unidos sejam o baluarte do bem, mas que o terrorismo internacional, especialmente o islâmico, é um mal muito grande que tem que ser enfrentado e derrotado. Da mesma forma que o tráfico de drogas.

Nós ateus, julgamos cruel e injusto que alguém passe a eternidade sofrendo no inferno. Mas apoiados em que padrão julgamos algo injusto?‎

Justiça não tem nada a ver com existência de algum Deus. Inclusive porque pode ser que, existindo, ilo não seja justo (ilo é a terceira pessoa do singular do pronome pessoal reto no gênero neutro - invenção minha). A justiça, em tese, se baseia na verdade e na moral que, por sua vez, novamente em tese, se baseia na ética. Em suma, fazer justiça significa, simplesmente, conceder a cada um o que merece por seus atos e comportamentos, que, pautando-se em uma moral que seja ética, sejam de modo a promover o bem. Então é justo recompensar o bem e punir o mal. Mas a punição tem que ser proporcional à maldade. O inferno seria uma punição infinita e não há maldade nenhuma que seja infinita.

O que acha da construção do novo World Trade Center? Não acha que estão cometendo o mesmo erro do passado,de querer demonstrar grandeza,em cima de um fato que nos marcou tanto?‎ .

Acho válido sua construção. E acho que os Estados Unidos têm o direito de demonstrar sua grandeza, como a França com a Torre Eiffel ou a Itália com a Basílica de São Pedro ou a Inglaterra com seu Parlamento, a Rússia com o Kremlin e assim por diante. Inclusive, acho bom que se mostre que não se teme o terrorismo. Se derrubarem essa se constrói outra.

"Ninguém vai fazer nada hoje" é uma frase que me deixa confuso, pois pra mim ela parece afirmar que todos vão fazer algo, ou que é errado dizer que alguém ficará sem fazer algo. Estou certo?‎

Acontece que a linguagem corrente nem sempre é lógica. Não tem ninguém, logicamente, significa que tem alguém, mas na linguagem significa que não tem pessoa alguma. Da mesma forma "ninguém vai fazer nada hoje", logicamente, significa que todos fão fazer algo, mas na linguagem significa que todos não vão fazer coisa alguma. Por outro lado, fenomenologicamente, é impossível não se fazer algo se se está vivo, pois, pelo menos, há que se estar respirando e, se não se estiver dormindo, pensando. Mesmo dormindo está-se fazendo algo, isto é, dormindo.

Prof, eu tenho uma dúvida um pouco idiota. Como é feito o fazer científico(se é que possa ser chamado de ciência) na disciplina de história? Principalmente em momentos tão remotos como os da antiguidade. Ou ainda mais que isso, nos períodos pré-paleolítico.‎

Existem muitos modos do fazer científico e o modo peculiar da história não deixa de ser científico por ser diferente do modo das ciências exatas e biológicas. Investigar o que aconteceu em algum lugar em certo tempo passado é um procedimento científico, inclusive de extrema complexidade, pois consiste no cotejo de indícios provenientes de relatos documentais, arqueológicos, antropológicos e outros para que se possa de toda essa teia, extrair conclusões, sempre provisórias, sobre o que, de fato, deve ter ocorrido. De certa forma isso é semelhante ao método da geologia e da paleontologia, que, absolutamente, não são ciências humanas.

Hermes Trismegistro descreve Deus: “Deus não é a razão, mas o fundamento dela; não é o alento, mas o fundamento dele; não é a luz, mas o fundamento dela. Entenda: Deus é o que não pode ser descrito.”‎

Então é um conceito vazio. Tudo pode ser descrito. Aliás, ele está descrevendo Deus como o fundamento de tudo. Isso é uma descrição. Só que há que se verificar se existe mesmo um fundamento para tudo. Será que existe? Em meu entendimento não. Como saber? Só porque se acha que exista ou não exista? Não há, por enquanto, argumentos que mostrem que há um fundamento para tudo. Ou que não haja. Também não há razão para se dizer que não se pode saber se há ou não há. Daí a única posição razoável a respeito é duvidar. É o que faço. Não sou gnóstico nem agnóstico. Sou cético. Mas acho que deus não existe. Só não posso garantir.

Teólogo e mentiroso é a mesma coisa. Ele estuda as mentiras como elas são. Nenhum deles jamais disse: “Ei! Isso aqui é mentira!”. Não confunda com estudiosos que buscam a verdade. Este trecho foi tirado do livro "Em Busca do Deus Javé", de Alfredo Bernacchi. Concorda ou discorda?‎

Concordo. Acontece que os teólogos partem do pressuposto de que as ditas "revelações" de sua religião sejam verídicas "a priori" e não as examinam para chegar sua veracidade. Inclusive, por exemplo, teólogos cristãos não admitem que possa haver uma teologia islâmica ou bramânica, pois entendem que os pressupostos dessas religiões sejam falsos e só os cristãos sejam verdadeiros. Ora, não há como aferir qual das religiões seja verdadeira a partir de suas escrituras ou da fé de seus seguidores, pois há pessoas de fé sincera em todas elas. E essas pessoas creem em fatos diferentes que, evidentemente, não podem ser todos verdadeiros, pois se contradizem. Então, o critério de veritação, necessariamente, tem que ser extrínseco às religiões, ou seja, tem que ser filosófico ou científico. E os teólogos não admitem a primazia da filosofia ou da ciência sobre a teologia. Fica-se pois num impasse insolúvel. Daí se poder considerar, de fato, que a teologia seja, toda ela, uma farsa. Se, para aferir se alguma assertiva teológica seja verdadeira, se precisar valer da filosofia ou da ciência, então, de que serve a teologia? Ela seria, apenas, um parte da Filosofia, no caso de se verificar que, verdadeiramente, existiriam deuses ou espíritos. Daí se poderia conceber uma disciplina que estudasse suas propriedades. Mas essa disciplina, certamente, não poderia se basear nas ditas revelações, e sim em verificações fáticas.

Professor, sobre o acaso. Ouço sempre nas aulas de Biologia, tanto teórica quanto prática, que tudo é ao acaso. A vida surgiu ao acaso. As mutações genéticas, recombinações e etc são ao acaso. Então, de acordo com o que o senhor disse logo a pouco sobre o acaso, isso está errado?‎

Não. O que está errado é considerar que o acaso seja um agente produtor de algo. Está certo dizer que algo aconteceu por acaso, mas isso não significa que houve algo, "o acaso" que produziu tal coisa. O que significa é que tal fato não foi produzido por nada. Aconteceu sem que houvesse algo que lhe provocasse a ocorrência. Isto é o que significa que foi por acaso. Mas há quem ache que acaso seria um agente. Esse é o erro.

A saúde brasileira enfrenta grandes desafios.Na sua opinião que atitudes devem ser tomadas para que acontece melhorias ?‎

A mais importante é acabar com a ganância dos médicos.

Vc aceitaria lição de moral de uma pessoa que teve uma vida desvairada?

Acho que o que uma pessoa diga não tenha sua validade aferida pelo modo como ela se comporta. Qualquer um que se comporte muito bem pode dizer muita besteira, bem como quem se comporte mal pode dizer algo muito sábio. Então, não vejo problema em ser admoestado por quem quer que seja, em relação ao que faço ou que digo, desde que essa pessoa apresente argumentos válidos para o que diz, independentemente do como se comporta.

em deus eu sei que vc não acredita, mais e no mal, pq msm não tendo muitas provas da existencia de ambos, ele se manifesta mais do que o bem.‎

O bem e o mal existem. Isto é patente. Mas não se tratam de entidades e sim de qualidades das ações. Todavia não acho que o mal seja manifestadamente mais abundante do que o bem. Não tenho registros estatísticos que me permitam concluir isso, mas penso que as ações benévolas são mais abundantes do que as ações malévolas. Incluindo aí não só ações voluntárias de seres conscientes mas ações involuntárias de seres inconscientes, como os fenômenos da natureza.

Pode existir algo mais perfeito do que o cerebro humano?

Claro que pode. Não conhecemos os seres que porventura existem em outros planetas e nem os que ainda existirão na Terra no futuro, produzidos pela evolução. Nem sabemos se outros animais, já extintod, eram mais inteligentes do que nós, como os Neandertais pode ser que fossem. O que se pode dizer é que, atualmente, o cérebro humano é o sistema mais complexo e capaz de realizações que se conhece. Mas não é perfeito. Também comete erros.

Ainda sobre Hitler.‎

Tenho múltiplos interesses, inclusive história. Mas não é o meu maior interesse, que está na cosmologia, depois física de partículas, depois neurologia, depois evolução, depois filosofia, depois música, depois pintura, depois literatura ... bem, história também está na lista e não em último lugar. Mas, só tenho 24 horas em cada dia e tenho que selecionar, deixando certos tópicos de lado. Dentre eles a biografia de tiranos, conquistadores, generais e esse tipo de gente. Quero saber da história das pessoas, da cultura, das estruturas sociais. Claro que a política e a economia também entram, mas secundariamente. As biografias que mais me interessam são as dos cientistas, dos artistas, dos filósofos, dos religiosos e dos líderes libertadores e não dos dominadores. É a opção pessoal que faço.

o republicanismo é sempre de direita?‎

Claro que não. Tanto o regime político democrático quanto a forma de governo republicana podem ser realizadas em concepções sociais quer da direita, quer da esquerda. Da mesma forma que os regimes autocratas e acratas, com a diferença que uma acracia, quer de direita ou de esquerda, não possui governo e, pois, não pode ser republicana.

Sua opinião sobre: Relacionamentos Poliamoristas.‎

Considero perfeitamente válidos, uma vez que todos os envolvidos estejam cientes e de acordo. Não vejo porque se restringir a possibilidade nem do sexo nem do amor apenas a um par de pessoas. É fato de que se pode amar, profunda, intensa e sinceramente a mais de uma outra pessoa. Então por que não se permitir a realização plena desses amores e se obrigar a fazer uma escolha que provocará tristeza? Isso implica, inclusive, em conceber a família como um grupamento que pode ser mais amplo, envolvendo mais de um pai ou mais de uma mãe, ou mais de dois pais ou duas mães, se os relacionamentos forem homossexuais. Para mim amor é algo que não pode sofrer impedimentos. Quanto mais amor houver no mundo melhor. Isso não significa que estou prescrevendo a poliginia ou a poliandria. Estou considerando que devam ser admitidas com toda a naturalidade, inclusive com todas as implicações sociais, econômicas e jurídicas. Ou seja, o relacionamento monogâmico tem que ser uma possibilidade e não uma determinação. Quem tiver mais de uma namorada ou mulher, mais de um namorado ou marido, tem que poder sair com todos na rua de modo perfeitamente natural e frequentar todos os lugares sem constrangimento. A lei tem que admitir os matrimônios múltiplos com todas as consequências em termos de pensão e herança, por exemplo. Os filhos têm que assimilar essa condição com toda a naturalidade e nutrir por seus múltiplos pais o mesmo afeto e respeito que nutririam para um conjunto de só dois. Acho que uma sociedade que admita isso seria muito mais feliz e honesta do que a atual, que, de modo hipócrita, só aceita, às claras, os relacionamentos monogâmicos, mas acoberta os casos extras, desde que não explícitos. É claro que isso só é possível com a mudança radical do conceito e do sentimento de amor de algo que envolva posse e controle para algo que seja libertário e exercido como doação, sem o menor resquício de ciúme. Não porque seja controlado, mas porque, realmente, não seja sentido. Isto é, cada um admitir, plenamente, que a pessoa que ama e com quem se relacione também possa amar e se relacionar com outra pessoa e todos serem uma família.

O senhor está aumentando e muito o meu vocabulário, obrigado professor!‎

Fico satisfeito em saber isso. Aliás, gosto de fazer uso de palavras mais específicas para os diferentes significados, exatamente para forçar os leitores a consultarem dicionários e, como isso, aumentarem seus vocabulários. É preciso largar a preguiça de lado e se dar ao trabalho de conferir o significado das palavras que são novas para a pessoa. Inclusive porque, atualmente, isso pode ser feito na internet mesmo.

Professor, estou no ensino médio e tenho uma dúvida (bem boba até) sobre Termodinâmica.Ficarei grato se você puder responder.Professor, pq eu não posso dizer que nos gases a energia cinética depende da massa e da velocidade, visto que Ec = m.v²/2? pq só posso considerar a formula ec = 3/2kt?‎

Corrigindo a resposta anterior em que não ficou grafado a velocidade média:
Mas é claro que a Energia Cinética depende da massa e da velocidade, mesmo nos gases, sejam perfeitos ou não. O valor 3/2kT apenas relaciona essa energia com a temperatura. Em verdade, pode-se entender que esse valor seja uma forma de definir a temperatura absoluta de um gás perfeito monoatômico se se igualar 3/2kT com 1/2mv², em que v seja a velocidade média dos átomos do gás.

Hitler era de fato cristão?‎

Pelo que consta, sim. Mas não sei se, em seu foro íntimo, realmente seria. E, na verdade, não tenho interesse em conhecer profundamente a biografia de Hitler, pois acho que ele não foi uma pessoa que mereça que eu perca tempo de minha vida me ocupando com ele.

o que tu acha de medicina?‎

Não posso deixar de achar algo maravilhoso, pois sem ela a mortandade da humanidade seria colossal. Não só ela evita que morramos por qualquer doença como nos possibilita que vivamos com uma qualidade de vida e saúde muito melhores. Só posso, pois, tecer loas.

Professor, tenho uma grande dúvida. Eu quero fazer Filosofia e poder dar aula. Mas ao mesmo tempo eu queria fazer pesquisa, "botar a mão na massa". Mexer com aquilo, laboratório e etc. Sempre quis fazer astronomia ( mas agora seria cosmologia), mas não sei se consigo ... O que eu faço Professor?‎

Acho muito bom que um cosmologista seja um filósofo, pois há grande conexão entre esses saberes. Um cosmologista, para começar, é um físico. Então, se for o caso de se dedicar aos dois ramos, é mais fácil fazer a graduação em física e pós-graduação em cosmologia, estudando filosofia por conta própria do que fazer a graduação em filosofia e estudar física e cosmologia por conta própria. O ideal, se se tiver disposição, tempo e dinheiro, é cursar física (bacharelado e licenciatura, para ser pesquisador e professor), cosmologia (no mestrado e doutorado) e filosofia também. Esta, inclusive, poderia ser cursada em cursos à distância, em paralelo com a física, se não se precisar trabalhar para se sustentar enquanto isso.

A fronteira entre "Ciência" e "Mito" é algo totalmente menos clara quanto você designa: na realidade, toda a Ciência é um Mito, e todo o Mito é uma Ciência. A Revolução Científica foi inspirada pela cosmologia esotéricas hermético-alquímicas, eqt q o cientificismo é uma crença protestante.‎

Não concordo com essa concepção. Mesmo que, em suas origens, a ciência tenha provido do mito, atualmente a concepção é completamente divergente. Nos mitos, os conhecimentos são baseados em relatos ancestrais que, algum dia, foram formulados por alguém, com base simplesmente em suas opiniões. A ciência, por sua vez, busca, exatamente, fugir das opiniões e se basear em interpretações verificadas factualmente. Não é um mito considerar que as explicações científicas é que representam a maior aproximação da verdade, pois isso é diretamente verificado. O cientificismo não é uma concepção científica e também não é protestante. Ele surgiu com o positivismo, que é ateu. É uma concepção filosófica, aliás, equivocada.

Nós estamos vivendo atualmente uma ditadura comunista? Nunca entendi essa citação que vi na internet

Claro que não. O Brasil é uma nação democrática e capitalista. Não é ditadura, não é comunista e não é socialista. Quem afirma isso, ou está usando propositalmente de má fé, ou não sabe o que está dizendo. Claro que a democracia brasileira tem problemas e que o capitalismo brasileiro tem uma ingerência estatal maior. Mas isso não chega a caracterizar um socialismo de estado ditatorial, como acontece em Cuba, por exemplo.

Interessante a sua comparação entre as ciências exatas, biológicas e humanas em outra resposta. Porém trabalhando com geografia, vejo esta, ao estudar a natureza, com uma aproximação muito maior com as exatas e as vezes com as biológicas, do que necessariamente com as humanas.‎

A Geografia e uma ciência mista, em parte exata, em parte biológica, em parte humana. A Geografia Física, ou Geociências é uma ciência exata e biológica e a Geografia Humana e Econômica é uma ciência humana.

Essa transfusão de genes, emprestado de outros animais poderiam ser realizado em cobaias humanas, se não quais fatores que impedem nos seres humanos de ser tornamos "mutantes" ou apenas mais evoluídos?‎

Tecnicamente isso poderia ser feito, mas não seria ético, pois o resultado é imprevisível e poderia levar, ou à morte do mutante, ou ao surgimento de um ser com sérios problemas. A engenharia genética ainda não tem formas de garantir o resultado de algum experimento genético. Isso é feito em vegetais e animais, como cobaias e, mesmo, já como procedimento comercial, no caso de vegetais. Até se chegar a um grau de segurança que permita se provocar uma evolução artificial em seres humanos vai demorar. Se bem que pode haver cientistas sem ética nenhuma que acabem fazendo isso.

Professor vi um artigo com o seguinte exemplo: uma aranha tem uma determinada toxina x somente usada na espécie dela, e também a toxina encontrada em uma bactéria x. Ouve uma transfusão de genes no passado, existem outros exemplos..‎ Off...

Não houve transfusão de genes. O que ocorreu foi um desenvolvimento paralelo e independente que, por coincidênia, levou a um mesmo resultado. A evolução das aranhas é muito posterior à das bactérias.

Professor, na Unicamp é " Instituto de Filosofia e Ciências Humanas (IFCH) " e na USP é "Faculdade de Filosofia, Letras e Ciências Humanas (FFLCH)". Porquê separam Filosofia das outras "Ciências Humanas"?‎

Porque Filosofia não é uma ciência. É um conhecimento não científico, advindo da reflexão sobre a realidade. Inclusive é um conhecimento metacientifico, pois busca, também, explicar as ciências.

Professor, recentemente discuti com um colega se os cursos de ciências humanas (sociais, história e etc...) eram de fato científicos. O que o senhor pensa a respeito?‎

São científicos sim. Mas as ciências humanas têm características distintas das ciências exatas e estas das biológicas. Não só os conteúdos são diferentes, mas as metodologias também. As humanas se baseiam mais em levantamentos do que em experimentos ou observações diretas, mesmo que possam usar esses recursos. Além do mais, dependendo do caso, não formulam uma teoria preditiva, mas uma teoria explicativa. Seu objeto de estudo, que são os conjuntos de seres humanos, não exibe um comportamento com regularidade suficiente para que se formulem teorias rígidas. Mesmo assim, é interessante que elas busquem, sempre que possível, adequar-se às formulações científicas, pelo menos das ciências biológicas, que se baseiam muito em resultados estatísticos. Isso vai lhes conferir uma maior credibilidade e evitar a presença das "escolas de pensamento" e a ausência do "corte epistemológico", que tanto mina a solidez de seus fundamentos.

Professor, recentemente discuti com um colega se os cursos de ciências humanas (sociais, história e etc...) eram de fato científicos. O que o senhor pensa a respeito?‎

São científicos sim. Mas as ciências humanas têm características distintas das ciências exatas e estas das biológicas. Não só os conteúdos são diferentes, mas as metodologias também. As humanas se baseiam mais em levantamentos do que em experimentos ou observações diretas, mesmo que possam usar esses recursos. Além do mais, dependendo do caso, não formulam uma teoria preditiva, mas uma teoria explicativa. Seu objeto de estudo, que são os conjuntos de seres humanos, não exibe um comportamento com regularidade suficiente para que se formulem teorias rígidas. Mesmo assim, é interessante que elas busquem, sempre que possível, adequar-se às formulações científicas, pelo menos das ciências biológicas, que se baseiam muito em resultados estatísticos. Isso vai lhes conferir uma maior credibilidade e evitar a presença das "escolas de pensamento" e a ausência do "corte epistemológico", que tanto mina a solidez de seus fundamentos.

Como pode a radiação corpuscular, como a radiação alpha, ionizar um átomo ou molécula? Como ocorre o processo?!‎

Ao atingir um átomo e colidir com seu núcleo, uma partícula alfa lhe transferirá energia, o que ocasionará sua excitação, isto é, a colocação de seus elétrons em níveis mais elevados de energia. Essa energia, em geral, é superior à energia de ionização do átomo, o que faz com que, não apenas os elétrons ocupem níveis mais elevados de energia mas, mesmo, sejam arrancados do átomo, ionizando-o.

Professor, o senhor já leu o livro Os Despossuídos da Ursula K. Le Guin? Se sim, qual a sua opinião sobre a visão da autora sobre a sociedade anarquista?‎

Sim, há muitos anos. Considero um excelente romance, além de conter uma fascinante descrição de uma sociedade anárquica. Mesmo sendo uma concepção particular de anarquismo, é ótimo para se ver que a anarquia é uma possibilidade concreta. Aliás, esse foi um dos livros que me levaram a abraçar a causa anarquista. Do mesmo modo que Servidão Humana, de Maugham me levou a decidir pelo ateísmo.

Olha esta manchete: "iPhone 5S é esmagado com pressão de 18 mil kg/m² em propaganda." Massa/área pode ser usado como pressão ?‎

O kg que está sendo mencionado, em verdade, é kgf, isto é, uma unidade de força que vale o peso da massa de 1kg na Terra, à altitude zero e latitude de 45°, onde o campo gravitacional é de 9,80665 N/kg. Ou seja, 1kgf = 9,80665N.

Por que suas provas não tinham limite de tempo?‎

Porque eu pedia para os alunos escreverem um capítulo explicando algum tema de modo que alguém, lendo, aprendesse. Podia-se consultar o que se quisesse, menos o colega. Mas a questão era individualizada. Cada um tinha a sua diferente. A prova começava às 14 horas e podia ir até as 23 horas, pois era quando a Universidade fechava. As turmas tinham poucos alunos, terminais do bacharelado em física, nunca mais de dez. Como eles iriam ser professores e pesquisadores, tinham que saber explicar, dar exemplos. Às vezes eu pedia deduções que os livros não mostravam. Nunca problemas numéricos, pois eu lecionava física teórica (quântica e relatividade). Geralmente eles levavam quatro horas para redigir o trabalho. Podiam sair para lanchar e ir ao banheiro quando quisessem. As aulas também não eram exposições e sim discussões sobre o conteúdo dos livros que, geralmente, eram em inglês. Os alunos tinham que, antes, ter estudado o capítulo que era destrinchado na aula, tintim por tintim. até ficar completamente entendido.

O sr se considera um "positivista"?‎

De modo nenhum. Considero o positivismo um equívoco, se levado às últimas consequências. Mas vejo que ele possui componentes valiosos, como a rejeição ao dogmatismo religioso. Só que não vejo que a filosofia deva ser rejeitada aprioristicamente como inválida, tanto em metodologia quanto em conclusões. Outrossim, não pode ser aceita incondicionalmente, sem verificação. Mesmo sendo uma pessoa plenamente convencida do valor da ciência na investigação da verdade, isto é, das explicações corretas para o funcionamento da natureza e da sociedade, admito que outras formas de abordagem do conhecimento também possam ser válidas, como a vulgar e a filosófica. Todavia, em caso de confronto entre elas, a científica tem primazia, em face de seus critérios rigorosos de validação. Não acho, como Comte, que se deva, apenas, buscar conhecer "como" tudo acontece, mas, também, "porque", mesmo admitindo que, em alguns casos, essa resposta não possa ser encontrada. Há casos, contudo, em que pode. Outro aspecto que discordo no Positivismo é a sua intenção de transformar suas concepções em uma espécie de "religião". Isso é bobagem e acaba levando a outro tipo de dogmatismo que é o "cientificismo". Nenhuma concepção filosófica pode ser considerada dogmaticamente. É preciso se ser cético, inclusive em relação ao ceticismo.

professor, eu devo buscar ser amado?‎

Ser amado é algo indizivelmente maravilhoso. Mas não é algo que se alcance por se buscar. O amor tem que ser espontâneo. Basta ser uma pessoa autenticamente amável, íntegra, cativante, que o brilho de sua personalidade, de seu caráter, de seu charme se espraiará e acenderá o amor das pessoas por você e, em algumas delas, não só o amor philia, mas o eros. E pode ser que, por algumas dessas, você também seja fisgado por eros e sinta atração amorosa. Então é só realizar esse amor. Mas tem que ser um amor doação e não possessão. Senão ele se deteriorará.

O que acha da Cleópatra? Você a admira?‎ .

Não. Uma mulher interesseira e intriguenta. Em verdade não gosto muito de reis e rainhas, exceto os que realizaram grandes benefícios para o povo e governaram com muita sabedoria. Conquistadores, então, eu abomino. Um rei bom, para mim, é o que acaba com a monarquia. Mesmo as monarquias constitucionais, para mim, são uma excrecência que não tem razão nenhuma de existir. A não vigorar a anarquia e se ter algum governo, ele tem que ser democrático e republicano para ser válido. De preferência, parlamentarista. E isso não é coisa moderna.

Ernesto,por quê alguns ateus falam que tem vontade de destruir o cristo redentor e no lugar,colocar um simbolo pagão ateu? Acha que é para implicar com os crentes ou são um bando de ignorantes intolerantes?‎ .

Não acho válido destruir o Cristo Redentor e nenhum monumento religioso de nenhuma religião em nenhuma parte do mundo, como o Khmer Vermelho fez com aqueles Budas Deitados no Cambodja. As religiões fazem parte do patrimônio cultural de um povo. Não é porque não se cultiva mais a religião grega que se deva demolir os templos religiosos delas na Grécia. Nem os templos egípcios, babilônios, assírios, maias, astecas, e todos os demais. Da mesma forma que, mesmo que o cristianismo e o islamismo acabem, não se devem demolir as catedrais, as basílicas, as mesquitas. Isso é, realmente, o cúmulo da ignorância. O Cristo Redentor já se tornou um símbolo do Brasil, como a Torre Eiffel da França e a Estátua da Liberdade dos Estados Unidos. Mesmo que seja religioso, também é histórico e cultural. Deve ser preservado. Observe que sou ateu e acho que as religiões devem ser extintas.

Ernesto,por quê alguns ateus falam que tem vontade de destruir o cristo redentor e no lugar,colocar um simbolo pagão ateu? Acha que é para implicar com os crentes ou são um bando de ignorantes intolerantes?‎ .

Não acho válido destruir o Cristo Redentor e nenhum monumento religioso de nenhuma religião em nenhuma parte do mundo, como o Khmer Vermelho fez com aqueles Budas Deitados no Cambodja. As religiões fazem parte do patrimônio cultural de um povo. Não é porque não se cultiva mais a religião grega que se deva demolir os templos religiosos delas na Grécia. Nem os templos egípcios, babilônios, assírios, maias, astecas, e todos os demais. Da mesma forma que, mesmo que o cristianismo e o islamismo acabem, não se devem demolir as catedrais, as basílicas, as mesquitas. Isso é, realmente, o cúmulo da ignorância. O Cristo Redentor já se tornou um símbolo do Brasil, como a Torre Eiffel da França e a Estátua da Liberdade dos Estados Unidos. Mesmo que seja religioso, também é histórico e cultural. Deve ser preservado. Observe que sou ateu e acho que as religiões devem ser extintas.

Não digo que sou um cético porque penso, mas também não sou religioso. Tenho uma idéia que aparentemente é válida , está é uma analogia e é a seguinte: tu fizestes uma compra em um supermercado X e este , porém , és longe de tua casa. O caminhão de entrega está indo levar suas compras...para tua casa , então ele pega uma avenida e é atingido por outro caminhão e faz vários giros em torno do seu CM e a massa para o bolo , o fermento , os ovos que tinham comprado se misturam e formam um bolo. Teoricamente e praticamente impossível. Essa é a questão de nossa criação , mesmo com.. ...o big bang há a necessidade de ter alguma criatura que seja superior a nós para encaminhas esses "ingredientes" para formar o big bang e finalmente nos criar. há fim de aprimorar meus pensamentos , tu , como um renomado professor e ateu queria saber oque acha dessa "tese".

Essa analogia não se aplica, pois a realidade atual do Universo não foi formada aleatoriamente a partir dos ingredientes elementares. A evolução, tanto cósmica quanto biológica é um processo por etapas, em cada qual as ocorrências são aleatórias mas para cada qual os resultados da etapa anterior é que são os elementos da nova etapa. Isso aumenta extremamente a probabilidade, como já mostrei neste texto:
http://www.ruckert.pro.br/blog/?p=819
Então, aleatoriamente, surgiu a inflação. Esta criou ondas gravitacionais que levaram ao surgimento de rarefações e concentrações de campo de matéria, que então se quantizaram nas condensações em quarks e léptons e suas antipartículas. Estas, por acaso iam se aniquilando, criando radiação e a radiação se transformava de novo em pares partícula-antipartícula, enquanto o espaço se expandia e o conteúdo esfriava, por transformação da energia cinética em potencial gravitacional. Daí começaram a se formar prótons e nêutrons e os outros bárions e híperons, Depois, esfriando mais ainda, se formaram átomos de hidrogênio e depois, de hélio. Estes foram os únicos formados no Big Bang. E tudo isso se deu por acaso. Com o desacoplamento, 380 mil anos depois, parou de haver aniquilação e criação de pares, ficando o Universo transparente e sobrando um resíduo de uma parte por bilhão de matéria em relação a antimatéria. Passados alguns milhões da anos, também em razão das flutuações de curvatura do espaço devido às ondas gravitacionais, começaram a haver regiões da maior e menor densidade, cujas primeiras deram origem às galáxias e, dentro delas, as estrelas e planetas. Tudo isso por acaso. No interior das estrelas se formaram os elementos pesados, até o ferro e, em suas explosões os além do ferro. O material ejetado nas explosões estelares formou as estrelas de segunda geração, como o Sol, em torno do qual se formaram planetas, dentre os quais a Terra. Tudo por coincidência, sem planejamento. A Terra, singularmente, formou-se de modo tal que propiciou o surgimento, também aleatório da vida, fenômeno singularíssimo no Universo, como se pode ver no livro "Sós no Universo?" de Ward e Brownlee. E, tendo surgido a vida, esta evolui por mutações aleatória e pela seleção natural dos mutantes mais bem adaptados ao ambiente externo e interno, (continua)É interessante observar como as estruturas filamentosas de aglomerados de galáxias no Universo em larga escala se assemelham a uma figura de interferência de ondas em uma cuba de ondas, considerada tridimensionalmente. Isso mostra as regiões de concentração e rarefação de matéria definidas pelas ondas gravitacionais.

A hipótese da inflação cósmica foi confirmada? http://revistapesquisa.fapesp.br/2014/04/24/tremores-big-bang/

Não acho que já seja uma confirmação definitiva, mas é, sem dúvida, um grande reforço à hipótese da inflação, bem como uma confirmação indireta da existência de ondas gravitacionais. O próprio Big Bang, contudo, já havia sido confirmado pela simples existência da radiação de fundo e pela recessão das galáxias.

Por que temos gostos musicais diferentes?‎

Ora, porque cada um é diferente. Não apenas o gosto musical, mas o gastronômico, o pictórico, o comportamental, o literário, o cinematográfico e qualquer outro variam de pessoa para pessoa. Isso é bom. O mundo seria tremendamente chato se todos fossemos iguais.

Quanto tempo do seu dia o senhor dedica para leitura? Sua leitura é diversificada ou é apenas baseada em livros? Meus pais, por muito tempo, tiveram uma revistaria. Infelizmente falimos com a vinda da internet. O senhor acha que, um dia, a mídia física acabe sendo extinta para a virtual?‎

Hoje eu leio só umas duas horas por dia ou menos. Mas leio livros, revistas e artigos da internet. Leio muitas revistas. Compro várias. Não leio jornal e nem vejo notícias pela TV. Os livros eu costumo ler uns três ou quatro em paralelo. De assuntos variados. Geralmente um romance, um científico e um filosófico ou histórico. Gosto muito, também, de psicologia e neurociências. A média é de uns três ou quatro livros por més. Antes eu lia mais, quando não entrava muito na internet.

Em uma prova na universidade, quis demonstrar minhas respostas. Eu provei as resoluções com rigor matemático. Demonstrando formalmente as respostas, gastei todo o meu tempo em metade da prova. Não fiz a outra metade, mesmo sabendo de tudo, por falta de tempo. A nota será menor q 5. Falo com o prof?‎

[Cont. sobre a prova]. Eu sabia de tudo. Quis responder além do que foi pedindo, demonstrando com rigor matemático as questões. Isto me deixou sem tempo e só deu p\ fazer metade das questões. Minha nota será menor que 5 (de 10), mesmo sabendo tudo. Foi um erro? Devo tentar conversar com o professor?
Você deve conversar. Mas eu acho que não vai adiantar. De qualquer modo, quando uma prova tiver limite de tempo (as minhas do bacharelado em Física eu dava sem limite de tempo) é bom fazer uma média do tempo de cada uma resposta e não desenvolver demais, para concluir no tempo.

Prof,o que o senhor acha de coleções que tem como intuito "mastigar" o máximo possível o conteúdo? (for dummies,the complete idiot's guide,demystified...)‎

Valem como uma primeira abordagem, mas são insuficientes para quem deseje saber o assunto de fato. Mesmo assim costumo lê-las, consciente de que não bastam e, portanto, busco outras obras mais completas e profundas.

Há comprovações concretas sobre a existência de espíritos?‎

Que eu saiba, não. As supostas manifestações de espíritos não resistem a verificações procedidas cientificamente.

Professor, acha válido votar em um candidato mesmo sabendo , com certeza,que ele não será eleito ? Se achar , o voto se justificaria por achar o condidato mais competente, mas, se por algum 'milagre' ele for eleito, a falta de apoio não seria um fator 'desclassificatório' ?‎

Nunca se pode ter certeza de que alguém não vá ser eleito. Sempre voto nos candidatos que eu considero que, dentre os apresentados, seja quem eu queria que vencesse, independentemente de sua cotação nas pesquisas. Se se votar com base em pesquisas está se podando a chance dos piores colocados de se elegerem. Os fatores envolvem competência e probidade. Se nenhum as apresentar, não voto em ninguém, porque não quero que nenhum candidato corrupto ou incompetente vença com a minha colaboração.

Você está generalizando o motivo pelo qual pessoas seguem religiões, e isso, ao meu ver, é um erro.‎

Concordo. Mas é essa a razão maior para as pessoas que seguem religiões. Elas o fazem, não porque examinaram todas as religiões, refletiram a respeito e decidiram lucidamente por qual delas optar. Mesmo algumas conversões não se dão por exame e reflexão, mas por indicação de outras pessoas, que apresentam as vantagens da nova religião, sem que o convertido examine todas. No meu entendimento, na grande maioria dos casos, toda pessoa que fizer um exame abrangente, profundo e crítico das várias religiões e cotejar suas propostas explicativas com as propostas científicas, bem como estudar a história delas e comparar suas concepções teológicas, concluirá pela inadequação de qualquer uma á realidade do mundo. Não estou dizendo que conclua pela inexistência de Deus, mas pela impropriedade da proposta doutrinária das diversas religiões. Mesmo que haja quem proceda da forma reflexiva e lúcida de escolha religiosa, a esmagadora maioria segue suas religiões porque é a que o meio que a cerca também segue.

Acho que você errou de resposta heheheheh http://ask.fm/wolfedler/answer/111750701597‎

Não acho que errei. As pessoas seguem as religiões porque não questionam, isto é, fazem assim porque todo mundo faz assim, como os novos macacos. A escola tem que cortar esse modo de ser da sociedade e debater tudo para ver se se justifica ou não.

O vídeo a seguir pode ser usado para explicar as religiões hoje em dia? (sem contar o fator geográfico e tradição de pai para filho) https://www.youtube.com/watch?v=_8zA18LkPR4‎

Isso mesmo. É dever da escola questionar tudo, sem exceção. Não para rejeitar tudo, mas para aceitar o que deve ser aceito com conhecimento de causa e rejeitar o que não tem razão para ser aceito. Infelizmente a escola não faz isso, mas só se preocupa em adestrar os alunos em responder questões de exames. Isso é que é o cúmulo da burrice.

Do que é feito os sonhos?‎

Os sonhos são feitos do conteúdo armazenado na memória, adquiridos pelos sentidos e processados pelo cérebro. Esses conteúdos incluem imagens, sons, odores, sensações táteis e outros dados declarativos, bem como memórias procedurais, emotivas e de toda ordem. O cérebro processa esses conteúdos em razão das vivências da pessoa, de seus desejos, de seus recalques e constrói um enredo, nem sempre plausível realisticamente, mas que expressa situações que a pessoa concebe como tal. Essa construção é levada a um nível de semi-consciência que, se ocorrer o despertar da pessoa enquanto se dá, torna-se consciente e o sonho fica registrado podendo ser rememorado. Mesmo que isso não aconteça, a pessoa sonha. A privação do sonho, mesmo sem privação do sono, isto é, quando a pessoa é acordada mal um eletroencefalograma mostre que esteja sonhando, pode levar a graves consequências, inclusive à morte. Isso acontece, também, com vários outros animais de estrutura neurológica mais complexa, como os mamíferos.

Professor, não consigo imaginar como uma coisa possa não ter causa, quando o Sr responde sobre o big bang geralmente o vejo afirmando que há coisas que não precisam de algo anterior paracontecerem. Poderia exemplificar exemplos desse tipo?‎

Para que um evento aconteça é preciso que o sistema em que ele ocorrerá se situe em uma condição que possibilite que ele se dê. Uma vez isso posto, a ocorrência pode se dar espontaneamente ou provocada por uma interação que tal sistema sofra por parte de outro. Essa interação, quado existe, se chama a "causa" do evento ocorrido, que então é dito "efeito" dessa causa. Muitos eventos são efeitos de causas, especialmente os eventos globais que ocorrem com sistemas compostos de número muito grande de elementos, sendo o evento global ou macroscópico a ocorrência conjunta de um imenso número de eventos elementares, havidos com cada um de seus componentes. Por exemplo, a emissão de luz por um filamento de lâmpada é um evento macroscópico resultante do decaimento de um número imenso (10^18 = 1.000.000.000.000.000.000 = um quintilhão) de átomos excitados nele presentes. Os átomos ficam excitados porque se agitam com a passagem da corrente elétrica por ele. Pode-se dizer, então, que a corrente elétrica é a causa da iluminação do filamento. Todavia, para cada átomo individual, a excitação não é a causa, mas a condição para seu decaimento, com emissão de fóton. O decaimento é fortuito, podendo se dar a qualquer tempo após a excitação, ou nunca. Nada provoca o decaimento, mas ele só acontece se o átomo estiver excitado. O que permite é a condição, o que determina é a causa. Então o decaimento é incausado. Mas, estatisticamente, sempre haverá átomos que estejam decaindo, portanto o filamento, certamente, emitirá luz. Todavia, rigorosamente falando, é possível que a corrente atravesse o filamento, excite os átomos e nenhum decaia, portanto ele não se acenda. A relação causal, em nível macroscópico, é, pois, uma relação estatística. Pode haver causalidade microscópica também, o que acontece, por exemplo, com o laser (light amplification by stimulated emmission of radiation), em que a emissão de fótons não é fortuita, mas estimulada. Daí a sua coerência de fase e colimação. Outros eventos incausados comuns são a desintegração radioativa e a formação de pares de partícula e antipartícula por flutuação do campo do vácuo. Note que a concepção de evento incausado só apareceu a partir de quando a Física pode examinar o comportamento da matéria, dos campos e da radiação em nível subatômico. Até então se considerava que todo evento seria um efeito, porque era o que se observava, em nível macroscópico. Mas essa conclusão é um raciocínio indutivo e, portanto, não tem garantia de validade perene, sendo derrubado pela existência de um único contra-exemplo. Atualmente se vê que os contra-exemplos, inclusive, são mais numerosos do que os exemplos.

É possível aplicar o método científico nas ciências humanas de modo que se anule subjetivismos e idiossincrasias?‎

Sim. Não o método da Física e da Química, mas o da Biologia, Medicina e Agronomia, por exemplo. Tal método consiste no estudo estatístico dos fenômenos, submetido à análise condizente com as características do fato. Isso pode ser aplicado à sociologia, à política, à economia, à psicologia, do mesmo modo que, também, se aplica à meteorologia, à astronomia. Métodos estatísticos, quando bem aplicados, permitem conclusões de grande confiança. Claro que os experimentos ou levantamentos têm que ser feito e analisados com extremo cuidado, inclusive para discernir relações de causalidade de meras correlações. E isso é feito. Assim procedendo, os cientistas podem evitar a introdução de opiniões subjetivas e concepções idiossincráticas em suas explicações de fenômenos sociais, econômicos, psicológicos e outros análogos. Tais procedimentos, inclusive, em meu entendimento, devem ser aplicados à Filosofia, mesmo que esta não seja uma ciência, propriamente dita, mas uma "metaciência". Tal "Filosofia Científica" é que advogo seja praticada pelos filósofos. Isso não significa rejeição à reflexão, mas sim a submissão das conclusões tiradas por reflexão a um critério científico de validação. Com o uso desses métodos de validação se evitariam a existência de "Escolas de Pensamento", tanto na filosofia quanto nas ciências sociais, que lhes corroem a credibilidade, por apresentarem mais de uma explicação, e divergentes, para um mesmo fenômeno, que, obviamente, tem uma explicação só. O método científico permitiria decidir qual a correta, a ser aceita por toda a comunidade, com o abandono das concorrentes. E não é garantido que, para fenômenos diferentes, a explicação da mesma escola é que venha a prevalecer.

Como a religião se arrasta com sua cultura arcaica ainda nos dias de hoje?‎

As religiões são fundamentadas em crenças sobrenaturais, em especial quanto a divindades e espíritos, se bem que se pode ter crença sem religião. Essas crenças advém da baixa disseminação, pela população em geral, de conhecimentos filosóficos, científicos, históricos e teológicos e, principalmente, do espírito examinador, reflexivo, questionador e, até, contestador. Não que quem possuam esses atributos necessariamente não possua religião ou crença religiosa, mas quem não os tem, normalmente, terá essas crenças ou será filiado a uma religião, porque não encontra base para ser uma pessoa que afronte as concepções genericamente aceitas pela sociedade. Então se submeterá a elas, inclusive porque é isso o que os detentores do poder querem que aconteça, pois quem contesta as religiões, acabará contestando, também, o poder político, militar e econômico, contribuindo para a derrubada do "status quo" que garante às elites dominantes sua tranquilidade como dominadoras e exploradoras do povo. Para reverter essa situação o que se precisa é introduzir na educação formal, desde as primeiras séries do nível básico, o treinamento reflexivo sobre a realidade e o conhecimento amplo sobre concepções diferentes das vigentes no lugar, época e estrato social em que se está inserido. Isso é essencial para dar à pessoa, enquanto se desenvolve, uma abertura de espírito capaz de aceitar concepções e modos de ser variados, bem como discutir os valores aceitos em seu meio. Isso se aplica não só aos aspectos religiosos como também aos sociais em geral, como as concepções gâmicas, por exemplo, as econômicas, as políticas, as prescrições morais, os costumes familiares e tudo o mais.

Comente a seguinte citação: Johannes Scotus Erígena (século 9 º): "Nós não sabemos o que é Deus. O próprio Deus não sabe o que Ele é porque Ele não é nada. Literalmente Deus não é, porque Ele transcende ser..."‎

Essa é uma concepção de Deus. Como saber se ela é correta? E como saber algo assim existe ou não, não sendo um ser e nem um ente? (Ser é o ente que, de fato, existe e ente é algo substancialmente concebível, isto é, que poderia existir, mesmo que não exista, mas não seja apenas uma abstração, isto é, uma concepção puramente mental, impossível de existir, como um círculo quadrado - há abstrações que podem ser entes, contudo). Em meu entendimento isso é um palpite dele, isto é, uma "doxa" e não um conhecimento estabelecido, ou seja, uma "episteme". Há outras concepções de Deus, como um ser transcendente pessoal ou impessoal, ou como uma entidade imanente. Como testar qual das concepções é verdadeira? Acho que nenhuma o seja, isto é, que Deus, simplesmente, não existe, seja qual for a sua concepção. Mas isso, também, é uma doxa minha. Não conheço um modo de conferir caráter epistemológico à existência de Deus. Mas não acho que seja impossível fazer isso (tanto quanto à existência como à inexistência), como consideram os agnósticos. Ou seja, não sou gnóstico nem agnóstico. Sou cético.

Ernesto, levando em consideração todas as consequências, positivas e negativas, que a religião traz, você acha que seria melhor viver com ou sem religião? Sendo ateu, você defende a presença da religião ou considera sua presença desnecessária,podendo ser substituída por outra corrente de pensamento?‎

A religião é completamente desnecessária e, até, perniciosa. Tudo de bom que as religiões fazem pode ser feito sem elas, como a filantropia e a contenção moral. Isso pode ser feito pela conscientização humanística e filosófica, que tem que ser desenvolvida nas escolas. Ao invés de escolas dominicais, aulas de catecismo, escolas corâmicas e outras que tais, há que se ter escolas filosóficas em que, desde a infância, se aprende a educar o caráter, a compreender a vida e a viver harmonicamente em sociedade. A cultivar as virtudes e abominar os vícios. Isso tem que continuar depois dentro da escola, de modo a formar pessoas íntegras, valiosas, beneméritas, justas, éticas, compreensivas, solidárias. Com elas o mundo se tornará um lugar aprazível e feliz para se viver. Não é preciso haver religião.

o tempo existia sim antes da grande explosão,tempo é imutável.‎

Não existia não. Tempo não é imutável. Tempo é decorrente das alterações do estado do Universo, como sua expansão. E ele pode parar de passar, se o Universo parar de se expandir, toda a radiação chegar à frequência zero e todos os sistemas materiais atingirem seu nível mínimo de energia com a entropia global atingindo seu máximo. Então, nenhuma alteração ocorrerá com tudo o que existe, o que implicará na cessação da passagem do tempo. Por outro lado, é claro que, se não existia nada não haveria passagem de tempo. Não existir nada significa que não existe nenhum conteúdo físico (campo, radiação e matéria) e tampouco espaço vazio para caber algo e nem tempo em que nada evolua. Essa era a situação antes de haver Universo. Nem "antes" havia.
Leia isto: http://www.ruckert.pro.br/blog/?page_id=1866

o impedimento é lógico. "universo" (entendido como matéria) existindo sem causa contraria o princípio da causalidade. Uma Causa Primária postulada existindo sem causa faz sentido, por tal coisa ser necessariamente incriada por definição, diferentemente do universo.‎

Não existe nenhum princípio de causalidade. Isso é um grande equivoco de filósofos, como Leibniz que consideravam que todo evento fosse um efeito, em razão de seu raciocínio indutivo embasado nos eventos macroscópicos, que eram os únicos observáveis. Com o advento da observação de eventos microscópicos, em nível subatômico, pode-se constatar que há eventos que não são efeitos, isto é, são incausados. Logo, causa não é uma necessidade, mas uma particularidade. Por outro lado, não há como se postular, por definição, que algo seja incriado. Isso tem que ser objeto de verificação. O fato de se definir uma entidade com as características que se concebe para Deus, absolutamente não garante a sua existência, como falaciosamente considera o Argumento Ontológico de Anselmo. Quanto ao Universo, ele pode perfeitamente ter sempre existido. Seu surgimento é factual, isto é, algo que se verifica fenomenologicamente, mas não uma necessidade ontológica. O dito "Argumento Kalam", usado por Aquino para pretensamente provar a existência de Deus em seu "Argumento Cosmológico", defendido por William Craig, também não procede, como já expus em meus blogs (digite no Google: inurl:wolfedler Kalam e inurl:wolfedler Craig). Não há, portanto, nenhum impedimento "lógico", nem "ontológico", nem "metafísico", nem "epistemológico", nem "fenomenológico" para que o Universo seja eterno. Ele não é porque, factualmente, não é. Mas poderia ser. Além do mais, Universo não é concebido como "matéria". Universo é o conjunto de tudo que existe de natureza física. Isso abrange, espaço, tempo, campo, radiação, matéria, estruturas e ocorrências. O Universo poderia não ter matéria nenhuma e nem radiação. Seu conteúdo poderia ser apenas de campo. Isso depende dos valores dos parâmetros das leis físicas. Poderia não haver átomos e, daí, nenhuma estrutura material, como galáxias, estrelas, planetas e seres vivos. Como também poderia não haver radiação, como a luz. Nem por isso deixaria de ser um universo físico. O que não pertence ao Universo Físico seriam as entidades espirituais (sobrenaturasi), caso existissem, e as abstrações, que são conceitos formulados por mentes e que não existem fora delas.

Professor acha errado fazer sexo sem estar comprometido ' namorando '?‎

Não. Não é preciso compromisso para se fazer sexo. O que não se pode é dar a entender que se está comprometido para obter o sexo sem se estar. Se os envolvidos estiver de acordo em fazer sexo só por recreação, sem compromisso ou sem amor, podem fazer à vontade e tenham bom proveito. Sexo, amor, relação e compromisso são fatos distintos que podem acontecer em qualquer tipo de combinação entre eles.

Uma pessoa que foi fornecida de dados incorretos sobre determinado assunto insiste em afirmar que esses dados são factuais. Essa pessoa estará dizendo a verdade?‎

Se ela estiver convencida de que os dados são correspondentes à realidade ela não estará mentindo, mas se os dados, de fato, não corresponderem à realidade, ele não estará dizendo a verdade. Isto é, uma pessoa pode nem estar mentindo nem estar dizendo a verdade. A verdade é a correspondência entre a realidade e o que se diz a respeito dela e a mentira é a não correspondência entre o que se considera que seja a verdade e o que se diz a respeito disso.

"Antes do Universo existir não havia tempo" A mesma crítica poderia ser feita ao Big Bang.‎

A inexistência de coisas implica na inexistência de tempo, pois o tempo advém das alterações no estado das coisas. Mas note que poderia não haver tempo e haver coisas. Big Bang foi o fenômeno do início da expansão do espaço do Universo. Admite-se que o Universo, com seu espaço, surgiu, também, nesse momento, com o espaço já se expandindo, o que significa o seu estado sendo alterado e, portanto, com o tempo decorrendo. Antes disso, mesmo que o conteúdo existisse, com seu espaço (que podia, até, ser infinito), não decorria tempo, pois tal conteúdo e tal espaço estavam imperturbáveis, ou seja, o estado global do Universo seria estático. Todavia se considera que, antes do Big Bang, não só não havia tempo, como também não havia conteúdo e nem espaço que o contivesse. Mas pode ser que houvesse. A teoria do Big Bang não diz nada sobre o surgimento do conteúdo que preenchia o Universo e nem sobre o espaço que o continha. De modo que poderia haver Universo sem haver tempo. Assim como pode ser que o tempo venha a parar de passar e o Universo continue a existir, sem tempo. A crítica que procede é, pois, sobre não existir Universo se não houver tempo. Isso pode acontecer (existir Universo sem tempo). Mas não pode haver tempo sem existir Universo.

"considerar que possa existir um Deus incriado mostra que não é preciso que tudo tenha uma causa. Então porque não supor que o próprio Universo tenha surgido sem ter sido criado?" Faz sentido uma Causa Primária ser incriada; o Universo, não.‎

Exatamente. Se Deus pode não ter causa, por que o Universo não pode? Isso não significa que ele seja Deus. Ou por que não pode ter sempre existido? Mesmo que esse não seja o caso, não há impedimento de que fosse.

Prof, faço física bacharelado mas não gosto das disciplinas de programação e também não me identifico com o turno das aulas pois sou extremamente vespertino e as aulas do bacharelado são de manhã. Seria uma boa opção migrar para física licenciatura (sem programação e aulas a noite)?

Vai depender do que você vai querer ser. Se quiser ser professor do Ensino Médio, então faça a licenciatura. Se quiser ser físico ou professor de Ensino Superior, tem que fazer o bacharelado, pois terá que fazer mestrado e doutorado também. Mas você poderá fazer a licenciatura e depois completar com o que ficou faltando do bacharelado para fazer o mestrado e o doutorado. Quanto a programação, atualmente é difícil ser um físico sem entender de programação. Logo, mesmo não gostando, vai ter que aprender. Por outro lado, nada impede que você mude seus hábitos e passe a acordar cedo. Para o aprendizado é melhor ter aulas pela manhã, estudar à tarde e descansar à noite.

acredita, ainda, que seja possível em algum dia o mundo ser um lugar melhor?‎

Claro que sim. E nem vai demorar muito. Em poucos milênios já se chegará lá, segundo penso.

Ernesto, recentemente eu dei uma opinião sobre o consumo de carne animal (que não seja humana) e, depois disso, chamaram-me, em outras redes sociais, de insensível e outros nomes que não vem ao caso. Na sua opinião, eu falei algo demais? http://ask.fm/PoeProust/answer/111599744312‎

Concordo com o que você disse. Não acho que comer cachorro seja imoral e porco não. Acho que a questão tem que ser debatida desapaixonadamente. Biologicamente somos onívoros, isto é, nosso trato digestivo é capaz de digerir carne. Portanto não somos veganos por natureza. Mas pode-se concluir que seja cruel matar um ser senciente para comê-lo. Ainda não cheguei a uma conclusão sobre isso e, por enquanto, como carne. Mas não humana.

Professor um TCC pode ser digitado?‎

Pelo que me consta sim. Não acho que se exija que seja manuscrito ou datilografado. Mas, para entregar, penso que se precise imprimir, mesmo que se entregue o documento em forma de arquivo também. Cada instituição possui as suas regras de apresentação que é preciso consultar. Aliás, acho, até, que se exija que seja composto em algum editor eletrônico de texto, o que, para mim, é um absurdo, pois entendo que seja perfeitamente válido entregá-lo datilografado ou manuscrito em papel almaço, com as figuras traçadas à mão.

Professor, o que você acha do livro "A mentalidade anticapitalista"?‎

Não li. Mas adianto que não aprecio as concepções de von Mises. Mesmo assim me inteiro delas, bem como as de Ayn Rand e os outros anarco-capitalistas, como o Rothbard e os nossos Rodrigo Constantino e até o Dâniel Fraga.

Suponhamos um infinito número de pedras no espaço infinito. Enumero as pedras com os números reais. Faço uma união bijetora entre o conjunto das pedras ímpares e o conjunto total das pedras. Como pode? A parte não deveria ser menor que o todo e tornar tão ligação um-a-um impossível?‎

Não, porque os conjuntos infinitos possuem propriedades não gozadas pelos finitos. Uma delas é ter uma cardinalidade igual à de alguma parte própria de si mesmo. Ou seja, parte do infinito pode ser igual a ele mesmo. Assim a cardinalidade dos pares, dos ímpares, dos naturais todos, dos inteiros todos (incluindo negativos) e, até, dos racionais, é a mesma. Note que não se pode falar do número dos números inteiros, porque isso não existe. Leia este trabalho:

O garoto já namora , mesmo assim afirma gostar de você, o que o senhor pensa sobre?‎

Isso pode perfeitamente acontecer. Não significa que ele não goste da namorada. Ele pode, simplesmente, gostar também de você. Se vocês três se entenderem, ele pode ter duas namoradas.

Acha que todos nós um dia, em algum ponto de nossas vidas, seremos humilhados ou nos sentiremos assim por outra pessoa? Por quê?

Não é verdade. Pode ser que várias pessoas sejam humilhadas alguma vez na vida, mas isso não é uma necessidade, de modo algum. Muitos podem, tranquilamente, passar a vida toda sem nunca serem humilhados. Não vejo de onde se pode concluir que isso seja fatal para todo mundo. Não há razão para tal.

O amor é perigoso?‎

De modo nenhum. Pelo contrário. É ótimo. Quanto mais se amar, a quanto mais pessoas e mais intensamente, melhor. O mundo repleto de amor será um mundo muito melhor e aprazível, Não se pode temer e nem restringir o amor. O que é perigoso é, justamente, o desamor, o ódio, a indiferença, Isso é que tem que ser combatido.

O egoísmo humano é inato?‎

O ser humano é potencialmente capaz de ser tanto egoísta quanto altruísta. Há quem seja geneticamente mais tendente ao egoísmo ou ao altruísmo. A expressão dessas tendências, contudo, se dará em função da interação da pessoa com o ambiente, em sua história de vida, desde a tenra infância. Principalmente em razão da educação que receba. Como o egoísmo, mesmo podendo ser vantajoso para a pessoa, é nefasto para a coletividade e, em última análise, é melhor viver em uma sociedade feliz do que cercado por hostilidades, Então é importante que o processo educativo promova a formação do caráter e da personalidade, desenvolvendo o altruísmo e combatendo o egoísmo.

A ética pode ser modificada pela moral?‎

Não. A ética se refere ao que seja certo ou errado de acordo com critérios filosóficos do bem e do mal, em relação às ações procedidas de forma consciente e livre. Ou seja, é o que se deve ou não se deve fazer para que o bem prevaleça. A moral é o que seja permitido, proibido ou prescrito, também em relação a ações, mas em função do que um grupo considere que o seja, em cada época, lugar ou estrato social. Idealmente a moral teria que ser ética, mas nem sempre o é. Ela pode, e muitas vezes o faz, refletir o interesse de grupos dominantes em relação ao que desejam que outros grupos façam. Assim há prescrições, proibições ou permissões morais que não sejam éticas, bem como ações éticas que a moral não permita ou não prescreva. Jamais, contudo, algo que a moral determine será, por essa razão, ético ou anti-ético, mesmo que possa o ser. Mas o critério ético não é o de ser conforme à moral. Três critérios sobressaem para a eticidade de uma ação: sua universalidade, isto é, que possa ser erigido em um imperativo, sua utilidade na promoção da maximização da felicidade para o maior número de seres e sua reciprocidade, isto é, que seja algo que deseje-se para si mesmo. Todavia é preciso analisar cada caso, face suas circunstâncias, para lhe conferir um valor ético. É importante frisar que tanto a ética quanto a moral concernem às ações e não aos seres.

Ontológicamente, o que é o vento?‎

Vento é ar ou outro gás em movimento.

Vc acusa Dilma de se entregar à fisiologia do presidencialismo de coalizão através da repartição de cargos, + esquece q sem isso não se governa o Brasil. O único presidente (Collor) q se negou a fazê-lo foi defenestrado do cargo. Dilma e PT propuseram reforma politica, mas o congresso ñ quer.‎

Isso é que tem que ser revertido por um ou uma presidente de peito mesmo. Enquanto essas práticas políticas nocivas não forem defenestradas não se conserta o Brasil. Tem que enfrentar a classe política corrupta e mudar a visão de política equivocada que se tem. Mesmo que se seja impedido. Mas se isso for sendo insistido sem esmorecimento, vai acabar. Não se pode desistir de consertar o mundo porque não se conseguem com poucas tentativas. Há que se insistir, pelo menos, 49 vezes, ou melhor, 490 vezes.

Ernesto, preciso da sua ajuda. Estou amando duas pessoas, e me vejo obrigado a escolher apenas uma. Quais critérios devo utilizar?‎

Você pode não escolher e amar as duas, por que não? Converse com elas. Quem sabe elas concordam. Acho que seria o melhor para todos os envolvidos. Se isso for impossível, é triste. Mas a escolha, para mim, recairia sobre quem reúne mais afinidades, exceto se a outra pessoa é aquela por quem se tem mais atração e se está mais apaixonado.

O que acha da idéia dessa escritora de "refazer" a obra com outra linguagem: http://www1.folha.uol.com.br/colunas/cidadona/2014/05/1445858-escritora-muda-obra-de-machado-de-assis-para-facilitar-a-leitura.shtml ?‎

Não gosto da ideia. Para mim, a qualidade da obra não está só no enredo, mas também na forma como é apresentada. De qualquer modo, para quem não curte literatura como arte e não como entretenimento, talvez essa seja uma forma de incentivar as pessoas a lerem os originais. Isso já foi feito com Shakespeare, por Charles e Mary Lamb, em 1807, na Inglaterra e se revelou uma forma de atrair leitores para as obras dele. Do mesmo modo que no caso da Divina Comédia e do Don Quijote.

Ernesto, "O bem sempre vence o Mal" é verdade ? Isso sempre acontecerá em todos os aspectos ?‎

Não. Pode ser que o mal vença o bem. Isso acontece em alguns casos e em outros o bem vence o mal. Não há nada que garanta ao bem ser vencedor só por ser o bem. O que é preciso é que a sociedade e cada um de nós se empenhe para que o mal não vença, em benefício de todos, pois o mal, quando beneficia, é só a alguns em prejuízo de muitos.

Ernesto, apesar de eu ser Pernambucano, não quero votar no Eduardo Campos e não vou votar no coxinha do Aécio... Estou fazendo algo de errado em votar na Dilma? Tenho gostado da administração do PT, especialmente nas políticas sociais e nas universidades (apesar de algumas falhas aqui e ali).‎

Não voto na Dilma, apesar de ter votado no PT em inúmeras eleições porque ela se tornou fisiológica. Loteou os cargos politicamente, o que abomino. Não agiu com firmeza para defenestrar todos os petistas corruptos. Para mim a lisura ética é "conditio sine qua non" para a administração pública, não podendo ser contornada em favor de nenhuma outra vantagem. Nessa história de Pasadena, não posso aceitar que ela não sabia de nada. Se presidia o Conselho, tinha que se inteirar de tudo, por obrigação de ofício. Não tem desculpa o que foi feito. Não confio na honestidade dela. Se vir que o Eduardo também não é, deixo de votar para presidente, porque no Aécio eu não voto por discordar de suas convicções ideológicas e nem gostar de suas companhias.

O laureado do prêmio nobel Sr John Eccles demostrou que as mentes são entidades não- físicas e isso causou o colapso do materialismo científico.Como se sente ao saber que o seu menosprezo contra as verdades bíblicas pode-le custar muito caro.‎

O Prêmio Nobel de Eccles não tem nada a ver com essa concepção. Foi sobre o papel das sinapses e dos neurotransmissores na condução dos impulsos nervosos. Quanto a suas concepções dualistas, ele era completamente equivocado. Ele não demonstrou que mentes não sejam físicas. Isso era uma opinião dele. Mentes são ocorrências naturais sim. Claro que não são materiais, mas são físicas, pois são ocorrências que se dão com o conteúdo material e eletromagnético do cérebro e seus anexos, bem como de todo o sistema nervoso, endócrino e, de certa forma, de todo o organismo. Não tem nada de sobrenatural, como a suposta "alma". Quanto à Bíblia, que verdades ela apresenta? Muito poucas. Cientificamente é um fiasco. Teologicamente, um absurdo. E historicamente, muito falha. Só lhe resta de aproveitável, e nem tudo, os preceitos morais e os textos de qualidade literária. De resto, é uma grande obra de ficção.

Curso Engenharia numa universidade feral.Com meus interesses não acadêmicos (filosofia, artes e literatura) fica complicado fazer tudo e dormir bem. Ás vezes durmo 3 ou 4 horas p\ um dia produtivo. Compensa prolongar este ato por vários anos?Ou gastar a vida dormindo, sem conhecimento mas com saúde?‎

Há que se achar um equilíbrio, senão você vai morrer novo, por ter deteriorado sua saúde por falta de sono. Isso, inclusive, afeta, também, a capacidade cognitiva. Menos de seis horas por noite é funesto. Quem sabe umas treze meias horas. Para se dedicar à aquisição de conhecimentos (e habilidades, não se esqueça), compensa deixar outras atividades, como ver televisão ou recrear. E aproveitar os tempos de trânsito lendo ou ouvindo leituras.

Concorda que o Estado deve ser administrado por filósofos?‎

Não. O estado deve ser administrado por políticos. Mas os políticos têm que ser capacitados para isso. Há que se exigir um exame de suficiência política em conhecimentos e habilidades políticas, administrativas, jurídicas, filosóficas, sociológicas, econômicas, históricas, geográficas, lingúisticas, científicas. Como se fosse um vestibular. Além, é claro, de vencer as eleições.

Você já se decidiu em qual candidato vai votar este ano para presidência? Se sim, qual, e porque? Não se sinta obrigado a responder.

Ainda não. Estou examinando. Mas, a princípio, estou preferindo o Eduardo Campos com a Marina Silva. Em quem, inclusive, votei na última eleição. Todavia ainda posso mudar.

"A Administração move o mundo." O que o senhor acha dessa afirmação?‎ ...

Discordo. O que move o mundo é o trabalho, mesmo não administrado. Aliás, este é o mais importante em países como o nosso. A economia informal é que não deixa os brasileiros morrerem de fome. Inclusive a economia de troca e de doações. Não fora isso os pobres não sobreviveriam. E tudo isso acontece sem administração. Administração é bom sim. Mas não é essencial.

O senhor vota? O que acha do voto obrigatório?‎

Voto, porque acho que, não votando, eximo-me da responsabilidade pela condução do País. Todavia sou contra o voto obrigatório. Isso tem que ser uma questão de consciência. O povo tem que ser conscientizado a votar sem ser obrigado. Do mesmo modo que sou contra a proibição do uso de drogas e também sou contra o seu uso. Isso inclui o tabaco. Como também sou contra a obrigatoriedade da monogamia e várias outras restrições à liberdade. Mas sou a favor de restrições no caso de ações maléficas ao outro, à sociedade e à natureza. Também não sou a favor da escolaridade obrigatória. Quem não quiser, que não estude. Ou a obrigatoriedade de trabalhar. Porque alguém é preso por não ter trabalho? Ridículo! Se não quiser trabalhar, não trabalhe, ora.

Se VC descobrisse que há uma inteligência artificial coletando todas as informações de todos os equipamentos eletrônicos do país para um programa anti-terrorismo do governo - sabendo que só a inteligência vê os dados - você apoiaria ou seria contra?‎ Robert Teutônico Claro que eu seria contra. Inteiramente. A segurança não vale a falta de liberdade. O desafio é manter a segurança e a liberdade também. Mas esta, jamais, pode ser sacrificada àquela. há cerca de um mês5 pessoas curtiram isso

Claro que eu seria contra. Inteiramente. A segurança não vale a falta de liberdade. O desafio é manter a segurança e a liberdade também. Mas esta, jamais, pode ser sacrificada àquela.

Entendo seu ponto, mas continuo a discordar, uma vez que se um evento pode ser feito de maneira microscópicos pode acontecer de maneira macroscópica também, afinal, somos feitos de coisas microcópia. Eu sempre acreditei que tudo o que ocorre tem uma origem, e nada acontecesse por acaso‎

Inclusive, já cheguei a cogitar a possibilidade de prever o futuro baseado nisto, se de alguma forma pudêssemos mapear todas partículas do universo, e pegar sua carga de energia, talvez pudesses prever o que acontecerá no próximo instante. Porque tudo ocorre conforme o que ocorreu antes. Entende?
O que você está dizendo já foi dito por Laplace em 1814: « Une intelligence qui, à un instant donné, connaîtrait toutes les forces dont la nature est animée, la position respective des êtres qui la composent, si d’ailleurs elle était assez vaste pour soumettre ces données à l’analyse, embrasserait dans la même formule les mouvements des plus grands corps de l’univers, et ceux du plus léger atome. Rien ne serait incertain pour elle, et l’avenir comme le passé seraient présents à ses yeux ».
Mas ele estava enganado. Sua concepção era o determinismo newtoniano, fragorosamente derrubado pela Mecânica Quântica. O universo absolutamente não é determinista e nem necessariamente causal. Isso não é uma concepção filosófica, é uma constatação experimental, sobejamente verificada. O acaso age o tempo todo em miríades de ocorrências em todos os lugares do Universo. Tudo que não acontece por acaso é porque possui uma probabilidade muito concentrada que aparenta ser um fato determinístico ou causal. Mas, essencialmente, a realidade é probabilística. Os filósofos e cientistas anteriores a 1925 estavam equivocados em suas interpretações do mundo.

Entendo a sua visão sobre propriedade intelectual, mas não acha que a sua abolição desmotivaria a produção de conhecimento?‎

Só se a motivação for o lucro. Mas se o processo educativo mudar essa motivação para o progresso da humanidade, de forma altruísta, no meu entendimento, aumentará mais ainda. Isso é que é preciso mudar na mentalidade do mundo. Fazer tudo não pelo lucro mas pelo benefício global. A educação tem que incutir isso na mente dos jovens.

Dê sua opinião sobre: Propriedade Intelectual.‎

Como anarquista acho que a propriedade deva ser abolida, seja qual for. Mas isso é um processo evolutivo gradual. A primeira que deve ser abolida é a intelectual e já está sendo. Os softwares livres são um exemplo. Agora temos, também, a "economia de doação", em que a pessoa produz e doa sua produção (ou serviço), recebendo doações em retribuição ou não. Mas não vende nem troca. De minha parte, quero disponibilizar tudo o que produzir por escrito, desenho, pintura, palestra ou o que mais, de graça. Como os outros não me dão comida de graça, valho-me de meu trabalho para ganhar dinheiro para viver. Mas não tenho propriedade nenhuma. A única casa que tive vendi e doei o dinheiro para a família. Não tenho nada para deixar de herança. Só um Kadett 1995. Meu bem mais valioso é minha biblioteca, que doarei ao povo por meio de uma ONG ou uma fundação.

Gostaria de saber a sua opinião professor em relação as famosas pedras britânicas de Stonehenge, que estão posicionadas em um alinhamento megalítico perfeito, sendo que foi feito a mais de 3100 anos a.c. Existe a crença de alienígenas terem posicionado as pedras, qual a sua opinião?‎

O alinhamento das pedras de Stonehenge não requer a colaboração de alienígena nenhum. Os próprios humanos foram perfeitamente capazes de fazê-lo pela observação astronômica, do mesmo modo que as pirâmides do Egito e outras edificações da antiguidade e da pré-história. O cérebro daqueles humanos tinha exatamente a mesma capacidade que o atual. O que eles tinham é menos conhecimentos. Mas o suficiente para observar as estrelas e perceber a regularidade de seus movimentos anuais no firmamento.

Nunca me interessou essas profissões liberais, nem o dinheiro. Tudo o que eu gosto é de música (popular), e se eu cursar ensino superior um dia, gostaria de cursar jornalismo.Mas não tenho a mínima ideia de como é a universidade, e o ensino médio já é um tormento pra mim...‎

Os cursos de graduação são mais exigente do que o Ensino Médio, sejam quais forem, exceto se a faculdade for muito fajuta mesmo. Em geral, é mais fácil entrar para a faculdade do que sair dela. Você precisa superar esse tormento para aguentar uma graduação. Quem, realmente, não goste de estudar, é melhor não fazer curso superior, porque tem que estudar muito mesmo. Bem mais do que o Ensino Médio.

Gosta de Minas Gerais? Já visitou Belo Horizonte ou alguma cidade histórica daqui?‎

Claro. Moro aqui desde 1951 e só saí por dois anos para fazer mestrado no Rio, em 1979/80, Morei 24 anos em Barbacena e 38 anos em Viçosa. Trabalhei 4 anos em São João de Rei e 2,5 anos em Juiz de Fora. Minhas irmãs moram em Belo Horizonte e eu já passeei várias vezes em Ouro Preto e outras cidades históricas que ficam perto de Viçosa.

"Há escolas que são gaiolas e há escolas que são asas." O Senhor concorda ? Não tenho a mínima motivação para querer estudar. Não acho que a escola ajuda a desenvolver minha inteligência, acho que a escola me doutrina. Aprendo muito mais lendo livros em casa, por vontade própria, e na internet.‎

Sim, e entre elas todo um espectro de gradações. A motivação para estudar tem que vir de dentro de si mesmo, destarte o que a escola seja. Porque é estudando que se adquirem conhecimentos e habilidades para se inserir satisfatoriamente no mundo. Se a escola atrapalha, que se supere a escola e se estude, apesar dela, visando o saber valioso e não só o convencional para passar de ano. Quem se contenta em só estudar o suficiente para ser aprovado é medíocre. Mas a escola tem a obrigação de estimular a gana por conhecimento e aquisição de habilidades. Esse é o seu papel, ao lado da formação geral para a vida. Muitas não o fazem por incompetência, negligência, ganância ou propositalmente, o que é pior. Claro que os professores são mal pagos, mas isso não é desculpa. Têm que ser bons professores e, ao mesmo tempo, lutar por melhores salários. Mas que esses melhores salários sejam acompanhados por uma avaliação meritocrática consequente, isto é, os incompetentes e negligentes sejam demitidos por justa causa. O progresso de nosso país vai depender da qualidade da educação e ela só vai acontecer quando o salário do magistério seja não somente justo, mas atraente, de modo que quem tenha capacidade para ser médico, engenheiro ou advogado prefira ser professor porque ganha mais. Isso mesmo, Um salário de uns dez mil reais por mês em dedicação exclusiva para um professor do Ensino Médio. Vejam isto:

http://www.ruckert.pro.br/blog/?p=4538
http://www.ruckert.pro.br/blog/?p=4518

Perfeito professor, e o que o senhor pensa sobre o SUL do país, numa visão econômica, social e em relação aos outros estados do Brasil. Na sua opinião, eles são tidos realmente como "superiores" ou é apenas amor demasiado que faz com que exaltem sua terra?‎

Se algum estado brasileiro puder ser chamado de superior, em relação ao conjunto de índices econômicos e sociais, seria São Paulo. Mas não acho que isso proceda. Há aspectos em que alguns são melhores e outros em que outros é que o são. Claro que há um grupo que se destaca e um grupo que é pior. De modo geral os do Sul, acrescidos de São Paulo, Rio de Janeiro, Minas Gerais e Brasília estão entre os que apresentam melhores conjuntos de índices. Isso, contudo, não abaliza seus governos, pois é consequência do desenvolvimento histórico.

Professor,vc sabe me dizer sobre a lei de três estados formuladas por Comte,se tem alguma delas comum hoje na atualidade?‎

Trata-se de uma interpretação equivocada da realidade feita por ele. O positivismo não é uma concepção abalizada da evolução da sociedade, da mesma forma que o marxismo também não é. Ambos se centram em um aspecto em detrimento dos outros. Enquanto o positivismo se norteia pelo cientificismo o marxismo se baseia no historicismo e na economicismo, encarados de modo dialético. Ora, nem o cientificismo é o único critério de validação das explicações, nem a economia e, muito menos, a dialética. Esta é uma das formas de evolução, mas há outras, inclusive, na maioria das vezes as questões são polialéticas e não dicotômicas. Em verdade a sociedade humana é extremamente complexa para se poder descrevê-la por modelos simples como esses.

O que acha dessa frase de Theodore Sturgeon: "90% de todas as coisas são lixo"?‎

Certamente muito do que existe é lixo. Mas não sei de onde ele tirou a informação de que seja 90% e não 87% ou 93% ou outra fração. Para mim é bem menos.

Por que pessoas que estudam muuuuuuito acabam ficando '' meio loucas '' ?‎

Não ficam não. O que acontece é que as pessoas que estudam muito o fazem porque é isso que elas gostam de fazer. E as pessoas que não gostam de estudar acham que quem goste é maluco. Puro preconceito. Se for pensar assim eu teria que dizer que quem goste de futebol é doido porque eu detesto.

Professor, como alguém dotado de personalidade e intelecto acima do tido como "normal", me responda: O que o senhor pensa das tradicionalidades do sul do país.‎

Depende. As tradições folclóricas são interessantes e belas. Mas têm que ser entendidas como folclores, isto é, algo que se cultiva em ocasiões específicas e não na vida cotidiana. Isso vale para o chimarrão, as danças e as vestes das festas. Mas não podem ser impostas a que não as queira cultivar. Tradições relativas a costumes comportamentais retrógrados e prejudiciais à felicidade das pessoas, como o machismo, não podem ser preservadas.

Eu acho errado acreditar que algo fez-se de origem aleatória. Vi que você acredita que o Big Bang foi originado de um evento fortuito, ao caso. Isso seria basicamente assumir que situações aleatórias podem acontecer em nossa volta. Como aparecer uma pedra magicamente na frente por exemplo.‎

Essa não e a noção de aleatoriedade correta. Mágicas não existem. Os eventos macroscópicos não possuem a aleatoriedade essencial dos eventos microscópicos. Mas podem possuir outro tipo de aleatoriedade que é o funcionamento caótico, em razão da multiplicidade de fatores envolvidos. Como se dá na previsão do tempo. O surgimento do Universo é um evento singular, pois se refere a uma transformação única da inexistência de qualquer coisa (inclusive de leis naturais) para a existência de tudo. Isso é perfeitamente possível de ser fortuito. Mas também podem haver eventos fortuitos a nossa volta que vão interagir conosco e, inclusive, influenciar o curso de nossa vida. Não se trata de mágica, mas de coincidências. A evolução do Universo como um todo é feita de inúmeras coincidências que não permitem nenhuma previsão do futuro. E não é uma questão de falta de informação ou incapacidade preditiva. O indeterminismo é um componente essencial do comportamento do mundo, bem como a presença de eventos incausados.

Você acredita que exista relação entre intelecto/notas altas ?‎

Sim, existe. Minha experiência docente de mais de 40 anos e uns 5 mil alunos me permite concluir que, em geral, estatisticamente, com poucas exceções, quem tira as melhores notas é quem é mais inteligente e mais intelectualizado, isto é, quem tem gosto por conhecimento, estudo, aquisição de habilidades cognitivas e que dispende, com prazer, mais tempo para isso.

"Tirar uma nota baixa em uma prova, não significa exatamente que o aluno não sabe a matéria". Concorda com essa afirmação, professor? Poderia expor sua opinião?

Sim, certamente. Mas tirar nota baixa em todas as provas de alguma matéria, estatisticamente revela que a pessoa não a domina. Por isso é que as escolas propiciam muitas avaliações ao longo do ano, pois, assim, o aluno pode se recuperar. Mas se a maioria da turma se dá bem nas avaliações e algum aluno não se dá, existe algum problema com ele, que precisa ser detectado e sanado.

Professor, como respondes com clareza as pessoas, gostaria de saber como o senhor consegui adquirir tamanha intelectualidade:‎

Desde criança que tenho uma curiosidade insaciável. Por sorte meu pai e minha mãe eram intelectuais e me supriam de respostas e de livros, que sempre devorei com entusiasmo. Meu pai entendia de história, geografia, filosofia e sociologia e minha mãe de ciências, matemática, música e artes. Em toda minha vida escolar sempre busquei saber muito mais do que era exigido na escola, não só em abrangência quanto em profundidade. Não sei explicar a razão. Isso é natural em mim. Me comprazo em estudar e fruo um prazer imenso em adquirir conhecimento e desenvolver habilidades. Então ser intelectual é algo totalmente natural em mim, para o que não faço esforço nenhum.

terça-feira, 3 de junho de 2014

Professor, você acha que uma pessoa muito sentimental e dramática pode mudar essas característica s dela? Se sim, como uma pessoa pode se tornar mais racional? e se não, como viver com isso? Eu sou uma pessoa muito sentimental, e isso me incomoda muito, já que qualquer coisa me machuca.‎

Pode. Mas ser sentimental não é ser irracional. Pode-se ser tanto um quanto outro. A questão a ser vencida é o melindre, isto é, a reação de se ofender com facilidade e se sentir magoado com ocorrências de menor monta. Para mudar esse modo de ser é preciso se conscientizar de que não se é melhor nem pior do que ninguém e que os outros não se importam tanto conosco como pensamos que importam. A maioria das ofensas que as pessoas melindrosas sentem não é intencional por parte de quem a faz. Mesmo quando seja, é preciso não se importar, considerando que, quem assim nos ofenda, não merece que tenhamos por ela a consideração de nos importarmos com o que ela faça. Dependendo do caso é preciso recorrer à ajuda psicológica para superar esse comportamento. Mas isso não significa que ser possuidor de um sensibilidade apurada seja algo ruim ou irracional.

As pessoas bonitas tem uma vida melhor do que as que são.Conseguem melhores empregos,são mais aceitas pela sociedade,não são discriminadas,são menos suspeitas.Diria até que é mais fácil obter a absolvição em crimes em alguns casos.Impossível não dizer que não é verdade.‎

Sim, isso é um fato, decorrente de um preconceito absurdo em relação à feiura, que associa beleza a bondade, justiça, honestidade, inteligência, educação e outras qualidades e feiura aos defeitos correlatos. Não é verdade que exista essa correlação. Pessoas belas podem ser más, burras, mal educadas e possuidoras de outros defeitos, como pessoas feias podem ser inteligentes, boas, educadas e possuidoras de outras qualidades. Erradicar esse tipo de preconceito é um objetivo que tem que ser perseguido pelo processo educativo, como outros.

Professor, que LIVROS para uma pessoa que não conhece nada sobre Pedagogia e Neuro pedagogia você indicaria? Pretendo ajudar, em um projeto, os alunos de minha ex escola interessados em estudar mais. No entanto, não sei de que maneira posso apresentar os conteúdos de forma clara e não massante.‎

Recomendo os três livros do Pierluigi Piazzi: "Aprendendo Inteligência", "Estimulando Inteligência" e "Ensinando Inteligência" (Editora Aleph). Além disso:
Inteligência - Everton Luiz Spolaor - Design
Mentes Brilhantes - Alberto Dell'Isola - Universo dos Livros
Mantenha o seu Cérebro Vivo - L.C. Katz & M. Rubin - Sextante
Deixe seu Cérebro em Forma - C.L. Gediman & F.M.Crinella - Sextante
O Cérebro Criativo - Shelley Carson - Best Seller
Inteligência Plena - R.J. Sternberg & E.L. Grigorenko
Esses são livros mais práticos.
Sobre neurociências recomendo todos os livros da Suzanna Herculano-Houzel e, mais teóricos, os de Steven Pinker e António Damásio.
Num nível aprofundado, para Pós-Graduação em Neurociências (especialidade médica) recomendo:
Neuroanatomia Funcional - Angelo Machado - Atheneu
Fundamentos da Neurociência e do Comportamento - E.R. Kandel, J.H. Schwartz & T.M. Jessell
em Bilhões de Neurônios? - Robert Lent - Atheneu
Neurociências - M.F.Bear, B.W.Connors & M.A. Paradiso - Artmed
Neurociência Cognitiva - M.S. Gazzaniga, R.B. Ivry & G.R. Mangu

Porque um bloco de gelo não derrete instantaneamente ao se colocar na lava?‎

Nada é instantâneo na natureza. O gelo requer absorção de calor para derreter e isso vai acontecendo da superfície para dentro. Então ele vai se transformando em água e essa água se aquece e entra em ebulição. Mas, por mais rapidamente que o calor lhe seja fornecido, o que é função da diferença de temperatura entre ele e a lava, enquanto estiver se fundindo, ele permanece a zero grau Celsius. Então sua fusão completa demanda um tempo, mesmo que seja bem pequeno.

PORQUE TEM MEDO DE DEUS, ATEU E UMA PALAVRA QUE NAO EXISTE, E MEDO DE DIZER QUE DEUS EXISTE MAS NAO CONSIGO PROVAR, NOS SOMOS A SEMELHANCA, SO QUE COM ALMA DO SATANAS TENTANDO TTIRAR O DIABO DE NOS MESMO,

Não tenho medo nenhum de Deus. Os cristãos é que cultivam o famigerado "temor de Deus". Sinceramente eu gostaria que Deus existisse e que eu tivesse uma alma imortal para que, depois de morto o meu corpo, eu pudesse manter conversas eternas com ele e perguntar tudo o que eu quero saber a respeito de tudo. Mas sei que isso é uma ilusão. Nem Deus existe nem eu tenho nenhuma alma imortal.

ALBERT EINSTEIN SE BASEOU NA BIBLIA PARA TEORIA MAIS COMPLEXA DA FISICA ATUAL, RELATIVIDADE, ENERGIA NUCLEAR, A BIBLIA ENTAO E UMA PIADA, VIVEMOS ENTAO COM MEDO DA PIADA, LEIA MAIS BIBLIA E PRESTE ATENCAO EM SUAS LETRAS, AMEM‎

Claro que não se baseou na Bíblia. De onde você tirou essa ideia estapafúrdia? Já li muito a Bíblia e, justamente por isso, é que me tornei ateu, dentre outros fatores.

Você fez aulas de piano? Por quanto tempo?‎

Sim, por três anos, começando aos nove. Depois parai porque entrei para o ginásio e não estava conseguindo conciliar os estudos escolares com a música. Mas continuei a tocar por gosto. Até que me casei e saí da casa de meu pai, onde tinha piano.

Não acho que ciume seja uma questão de cosmovisão. É mais ligado ao instintivo e sentimental. Conheço pessoas completamente racionais que quando estão apaixonadas perdem a compostura por conta do ciúme.

A civilização e a cultura surgiram, exatamente, para domar o instinto. Não controlar o conter o ciúme é falta de educação sim. Como não conter a raiva, não vencer o medo, não dominar a inveja, não refrear a cobiça, não superar a preguiça. Ou falar mentira, ser desonesto, injusto, aproveitador. Ou, ainda, ser malvado, cruel, implicante, chato, seboso, pernóstico, convencido. Quem se deixa levar por isso tudo, a não ser que possua alguma patologia, é porque não se educou suficientemente. E o processo educativo, tanto em casa quanto na escola, tem que, justamente, treinar as pessoas a viver civilizadamente, contendo esses vícios nefastos.

O que acha do "princípio da razão suficiente" de Leibniz, que defende que nada acontece sem que haja uma razão suficiente para isso?‎

Trata-se de um grande equívoco. Mas pode-se entender que Leibniz assim o tenha concebido como um princípio, pois, à sua época, só se tinha acesso a eventos macroscópicos, que exibem, na quase totalidade, uma relação de causalidade, ou seja, são efeitos. Eventos que não são efeitos, isto é, que não possuem causa, só se mostraram a partir do momento em que se teve acesso a eventos em nível subatômico e a partir de quando a mecânica quântica passou a descrever a realidade a partir de outros pressupostos, que não os da física clássica newtoniana, determinista e causal. A causalidade e o determinismo (que não são a mesma coisa) advém da concentração probabilística de eventos que envolvem um imenso número de eventos elementares, levando a densidade de probabilidade a uma ser uma distribuição do tipo "Delta de Dirac". Por exemplo: ao ser excitado, um átomo tem seus elétrons (pelo menos o mais energético) elevado a um nível superior, que é uma condição para que decaia para o nível em que antes estava. Mas esse decaimento é fortuito, isto é, não provocado por nada. Quando ele se dá, há a emissão de um fóton de luz. Mas ele pode se dar a qualquer momento ou nunca. A informação que se tem é sobre a distribuição do tempo de espera para o decaimento, que é um decaimento exponencial, da qual se obtém a "meia vida", isto é, o tempo para que metade de uma coleção de átomos excitado tenha decaído. No caso de uma lâmpada incandescente, o filamento possui cerca de 10^18 átomos de tungstênio. Então a probabilidade de que alguma fração dos átomos decaia em um tempo bem pequeno é quase unitária. Portanto pode-se dizer que a excitação, provocada pela passagem de corrente elétrica no filamento, macroscopicamente falando, seja considerada a causa de seu iluminamento, mesmo que, para cada átomo, individualmente, não seja causa, mas condição.

Professor, como não ter ciumes? Principalmente da vida passada da pessoa ou de "colegas"? Há alguma fórmula pra isso?‎

Claro. Isso é uma questão de cosmovisão e, portanto, algo que pode ser alterado por reflexão a respeito da total falta de motivo para isso. Pense bem: qual a razão de ficar incomodado com o fato da pessoa amada já ter amado outras? Pense melhor ainda: por que a pessoa amada, mesmo nos amando, não pode amar, também, a outrem? Todo mundo é capaz de amar a mais de uma outra pessoa e isso acontece normalmente. Amor sincero, intenso e profundo. Mas as pessoas se vêem obrigadas pelo costume social a renunciar a outros amores simultâneos em favor de um só, em geral com sofrimento, pois gostariam de vivenciar todos. Então, por que não? Tudo de modo conhecido e consentido. A possessividade amorosa é só uma questão cultural. É imoral porque a moral estabelece que assim o seja. Mas não fere a ética, pois não é maléfica para ninguém. Então a moral é que tem que mudar. Por que se aceita que uma pessoa ame diferentes outras pessoas sequencialmente e não simultaneamente?

Qual a importância de aplicar a disciplina de Educação Musical no ensino corrente?‎

Enriquecer o cabedal cultural da pessoa pelo aprendizado da apreciação musical em razão do conhecimento que se tenha de música. Além de possibilitar que vocações musicais sejam despertadas. Não só música, mas artes plásticas, teatro, poesia devem ser objeto de estudo e prática pelos alunos e alunas da Educação Básica, visando o mesmo objetivo. E muito mais, como já disse em outras ocasiões.

A opinião dos físicos Neil deGrasse Tyson e Lawrence Krauss é que "tudo leva a crer que existe outros universos, embora não tenhamos como provar". Concorda com eles?

Não. Não acho que tudo leve a crer não. A suposição da existência de outros universos decorre de considerações relativísticas e quânticas. E esses dois casos conduzem a conceitos diferentes de universos paralelos. O relativístico advém da extensão maximal das soluções de Kerr-Newmann (http://pt.scribd.com/doc/221824251/Buracos-Negros). O quântico advém da interpretação de muitos mundos de Everett (http://pt.wikipedia.org/wiki/Interpreta%C3%A7%C3%A3o_de_muitos_mundos). Essa interpretação está bem descrita no livro "The Fabric of Reality" de David Deutsch. Para mim são possibilidades hipotéticas completamente carentes de confirmação e, mesmo, de plausibilidade. Entendo que este universo seja único e que as possibilidades quânticas não realizadas sejam, simplesmente, perdidas. Quanto aos outros universos relativísticos, eles não passa de soluções imaginárias.

Sr Rückert, o que supostamente o homem precisa para dominar a fusão nuclear?

Conhecimento e tecnologia, advinda desse conhecimento. A fissão nuclear já é domada, mas a fusão não. As propostas de fusão nuclear controlada envolvem o confinamento de plasma em um toro por campos magnéticos. Mas isso ainda é muito caro e não se consegue, por enquanto, uma geração significativa de energia. A dita "fusão nuclear a frio" revelou-se uma balela. Todavia as pesquisas estão em andamento. Veja isto:
http://pt.wikipedia.org/wiki/Tokomak

Qual a opinião geral do povo brasileiro acerca de portugal e dos portugueses?‎

Como sou neto de avó portuguesa (e avô austríaco) e convivi com tios portugueses, tenho um grande apreço pelo povo português, que, absolutamente, não é o que se diz que seja nas piadas. A questão é que os portugueses são muito mais lógicos e disciplinados do que os brasileiros, que são mais informais e espontâneos. Mas não são só os portugueses que são assim. Europeus em geral o são, exceto italianos e gregos. Franceses, ingleses e alemães são como os portugueses. E são todos muito mais sérios, sem perder a alegria. Os artigos da Wikipedia escritos em Portugal, na maioria, são bem melhores do que os escritos no Brasil. Brasileiros não gostam de levar as coisas muito a sério, de modo que zombam dos portugueses. Mas eu não gosto é desse modo brasileiro de ser. Esse modo de levar a vida na brincadeira, de não ter compromisso, de ser indolente. Sinceramente, não me identifico com isso e não me considero um brasileiro típico.

Por que alguns tipos de musicas conseguem nos deixar com raiva ou ate mesmo alegres?‎

A música age nos centros mais profundos do cérebro, provocando emoções. É capaz de acalmar e excitar, de inspirar amor e ódio, alegria e tristeza, euforia e tédio. Muito mais profundamente e de modo muito mais eficaz do que percepções visuais. Sugiro a leitura do livro "Alucinações Musicais" de Oliver Sacks (Cia das Letras). O único sentido mais primitivo do que a audição é o olfato.

Texto escrito pelo economista e presidente do Instituto Liberal, Rodrigo Constantino , para o jornal O Globo de terça, 29/04, defendendo o capitalismo: https://lh6.googleusercontent.com/-YY1Qatj9QyQ/U2A7kMUTmrI/AAAAAAAAAFg/b1nd0RVJOk4/w592-h577-no/Sem+t%25C3%25ADtulo.png Você poderia comentar?

Concordo em boa parte com o que ele diz, tanto que não sou socialista e sim comunista, o que é muito diferente. Socialismo é a forma marxista de se atingir o comunismo, que é totalmente equivocada. O comunismo tem que ser alcançado pela total exacerbação e pulverização do capitalismo, a tal ponto que todas as pessoas sejam capitalistas e ninguém seja empregado assalariado. Isto é, todos sejam sócios dos empreendimentos em que trabalhem. Um mundo sem empregos, só com empresários. Isso é o comunismo, isto é, uma sociedade em que tudo seja comum. A desigualdade tem que haver nos direitos, nos deveres e nas oportunidades para todos, independentemente de como seus pais aproveitaram as oportunidades deles. Mas as pessoas são diferentes em natureza, de modo que uns aproveitarão mais as oportunidades do que os outros. Isso é normal. Não se pode confundir, todavia, concepções esquerdistas com concepções socialistas. Esquerda e direita são mais concepções sociais do que políticas. A esquerda considera a necessidade dessas igualdades de que falei e a direita considera que elas sejam justas e necessárias. Ou seja, a direita considera que é preciso haver pobres e ricos e a esquerda de que não seja preciso, que todos possam ser ricos. Não que todos tenham que ser ricos. No socialismo todos se tornam empregados de um único patrão, o governo. No comunismo todos são patrões e ninguém é empregado de ninguém. Note que comunismo, socialismo e capitalismo são concepções econômicas e não políticas. Democracia, autocracia e acracia são concepções políticas. Esquerda e direita são concepções sociais. Pode haver várias combinações entre elas. Mas o socialismo não pode ser ácrato (anárquico), pois requer um governo. O capitalismo e o comunismo podem existir sem governo ou com governo democrático. Não consigo conceber um comunismo autocrático. Note que o que foi chamado de comunismo na antiga União Soviética e o que há em Cuba, não é, absolutamente, comunismo. É socialismo estatal autocrático. Isso é péssimo.

Como saber se Deus existe?‎

Não há como, face os conhecimentos atualmente disponíveis. Nada há que garanta a sua existência e nem a sua inexistência. Então tem-se que crer ou não crer. Todavia, mesmo não havendo evidências e nem provas a favor ou contra essa existência, existem indícios. E esses indícios são fortemente contra a sua existência. Já os listei em outra resposta. Digite no Google:inurl:wolfedler indícios deus


Por que é tao difícil acabar com alguem?‎

Não acho que seja difícil. Só não vejo razão para isso. Tanto não é difícil que assassinatos estão acontecendo com frequência no mundo. Certamente que isso pode acarretar a condenação do assassino, mas, pode ser que ele queira tanto a morte de alguém que não se importe de ser condenado por isso. Todavia a sociedade precisa coibir tais atos, para sua própria proteção.

Professor, poderia explicar a lei de Gauss ?‎

A Lei de Gauss diz, simplesmente, que a quantidade de campo elétrico ou gravitacional emanado por uma carga ou uma massa é proporcional ao total de carga ou massa dessa fonte. Matematicamente isso é expresso pela igualdade, a menos de uma constante, entre o total de carga no interior de uma superfície fechada e o fluxo do campo que atravessa essa superfície. O fluxo é uma medida da quantidade do campo, dada pela integral (soma de infinitas parcelas infinitamente pequenas) do produto de cada elemento de área da superfície pela intensidade da componente perpendicular de campo que o atravessa. A vazão de escoamento de um fluido e a corrente elétrica são exemplos de fluxos. Também há fluxo de radiação eletromagnética. No caso da vazão de um fluido e da corrente elétrica, o equivalente da Lei de Gauss é a "Equação da Continuidade".

Estou estudando para o ENEM e não tenho tempo para ler livros de interesse momentâneo, como neurociências, então pretendo faze-lo no momento de lazer/descanso. Recomendas esse tempo, apenas em finais de semana, depois da escola, quando? Quais suas recomendações gerais para os estudos?‎

Além do tempo de aulas é preciso dedicar, pelo menos, dois terços desse tempo para estudo todo dia. Isso deixa, ainda, um bom tempo diário para lazer, que pode ser usado para leitura de outros assuntos também. É bom ter um tempo de estudo mesmo nos fins de semana e feriados. Isso é como um pianista. Não pode passar um dia ser treinar, não importa o que haja. E o estudo tem que ser do assunto da aula, no mesmo dia da aula, antes que se durma. Mas ainda resta tempo para outras atividades. Basta não ver muita televisão e nem dormir demais.

Como o Universo é curvo se fora dele não existe espaço?‎

Pode ser que o Universo como um todo seja curvo, mas, ao que tudo indica, não é. Essa curvatura é intrínseca. Não é como a curvatura de uma superfície bidimensional imersa em um espaço tridimensional. Mas ela pode ser verificada, por exemplo, pela soma dos ângulos internos de um triângulo. Se for dois retos, o espaço é plano, se for mais de dois retos é positivamente curvo, se for menos de dois retos é negativamente curvo. Em um espaço bidimensional, como a superfície de uma esfera (considerada em si mesma e não como parte de um espaço tridimensional), a soma dos ângulos internos de um triângulo é mais do que dois retos. Num espaço tridimensional intrinsecamente curvo é a mesma coisa. O que se verifica é que, em grandes extensões, o espaço do Universo é plano. Em torno das concentrações de massa e energia, o espaço-tempo (e não apenas o espaço) é curvo e essa curvatura é que produz os efeitos dinâmicos gravitacionais. Se o espaço é plano, ele é infinito. Se ele for finito, é positivamente curvo. Um espaço negativamente curvo também é infinito. É interessante ler o livro "Hiperespaço" de MIchio Kaku.

Professor, estou a dias tentando resolver um equação com 4 cognitivas. Há como resolver sem usar "testes"? Ex: a + b + c +d = 20 (pode ser que a,b,c,d = 5) mas como ei de resolver esse problema sem ficar chutando valores? Procurei por tudo e não achei nada a respeito de cálculos.‎

Não existe solução para isso. O que se pode é estabelecer o valor de uma das variáveis em função das outras. Mas as outras podem ter qualquer valor, de modo independente. Ou seja, existem três graus de liberdade, sendo o quarto valor determinado pelos outros e pela equação. E note que os valores não precisam ser inteiros e nem positivos.

Como me situo no espectro político? Considero as desigualdades justas (se todos tiverem a mesma oportunidade, partindo de um mesmo ponto em comum onde o contraste é posteriormente gerado pelo extremo empenho de alguns, médio empenho de outros e preguiça de muitos).‎

Você é uma pessoa de direita. Mas isso é uma concepção sociológica e não política e nem econômica. Um direitista pode ser autocrata, democrata e acrata, politicamente falando. Economicamente ele não será comunista, mas pode haver socialismo de direita, bem como, naturalmente, capitalismo. O Nazismo foi um socialismo de direita.

Sociologia e Filosofia são importantes? Por quê? É mesmo necessário existirem tantos sociólogos e filósofos por ai?‎

Sim, Muito importantes. Porque são conhecimentos que permitem a pessoa compreender a si mesma e o mundo em que se insere, Os filósofos e sociólogos são necessários para ensinar isso às pessoas, para que elas sejam curiosas, examinadoras, reflexivas, críticas, contestadoras, reformadoras e possam consertar o mundo e encontrar significado para suas vidas. Mas não um significado ilusório, como as religiões fornecem. Além disso, os filósofos e sociólogos são as pessoas que estudam tudo isso e elaboram propostas para a melhoria das condições sociais e pessoais das pessoas. Para tal eles também têm que entender de política e economia (dentro da sociologia) de de ciência e psicologia (dentro da filosofia). Todo político e administrador, por exemplo, teria que passar em um teste de conhecimentos filosóficos, psicológicos, sociológicos, econômicos, políticos, históricos, administrativos, geográficos e científicos em geral. Senão não teriam competência para exercer o seu mandato. Além, é claro, de possuir caráter ilibado (especialmente probidade), criatividade e inteligência para elaborar propostas adequadas e muita disposição para trabalhar duro a fim de obter os resultados. Isso vale para toda atividade a que uma pessoa vier a se dedicar na vida.

Professor, é possível alguém cursar 4 faculdades sequencialmente? Pergunto isso pois vou cursar Economia e Música, mas além dessas, pretendo cursar Filosofia e Física.‎

Claro que é, desde que tenha dinheiro para se manter enquanto estuda. É possível estudar e trabalhar simultaneamente, desde que não se tenha a menor preguiça. E nem precisa trabalhar como empregado. Pode-se trabalhar por conta própria. Atualmente ainda não se obtém comida e nem moradia de graça, de modo que há que se ter rendimento. Mas pode ser que se seja rico e não se precise trabalhar. Seja como for, é possível estudar não só quatro, mas muitos cursos ao longo da vida. As faculdades não limitam a idade dos alunos.

Professor, os físicos são aqueles com maior tendência a descobrirem novas formas de energia?‎

Não só os físicos. Engenheiros também trabalham com isso. Qualquer pessoa poderia descobrir, mas esses dois profissionais é que teriam mais chance.

Professor, eu lhe perguntei uma vez sobre a expansão e o "lado de fora" do universo, você me disse que ele é como uma esfera, dois ponto que se distanciam, mas não ficam longe. Eu realmente não entendi muita coisa, é possível explicar de um maneira mais didática? Obrigado.‎

Há duas hipóteses. Se o Universo for infinito, como se considera, atualmente, que seja, então não há problema de se entender que não há lado de fora, pois ele se estende indefinidamente em todas as direções.
Outra possibilidade é que ele fosse finito. Mesmo assim, ele não teria lado de fora e nem uma fronteira. Como assim? Nesse caso ele seria fechado sobre si mesmo, isto é, se se fosse indo sempre para frente, dá-se-ia uma volta completa e se chegaria ao ponto de partida, vindo por trás. Não exatamente no mesmo lugar pois, enquanto se contornaria o Universo, ele iria se expandindo. Para entender isso é que se faz a analogia com a esfera. Considere que as três dimensões espaciais do Universo (largura, altura e profundidade) se reduzam a duas (tiremos a altura). Então nos tornaríamos seres bidimensionais que viveríamos sobre uma superfície e não poderíamos sair dela, nem para cima, nem para baixo. Mas poderíamos nos mover para frente, para trás, para a direita e para a esquerda. Sendo o Universo infinito, essa superfície é um plano, que nunca acaba. Se fosse finito, essa superfície seria a superfície de uma esfera. Estando sobre ela (e não existindo mais nada fora dela), se se fosse para frente, contorna-se-ia a esfera sem achar nenhuma barreira onde ela acabasse e se retornaria ao ponto de partida. Isso é o que aconteceria com as três dimensões do Universo, sendo ele finito. Ele não teria limite e não haveria nada fora dele. Mas ele estaria crescendo, como se, na analogia da esfera, ela fosse um balão que estivesse se inflando.

Espere, não sei se compreendi direito ainda. Tanto animais quanto vegetais originaram-se de bactérias e archeas, logo têm um ancestral em comum (considerando-se que apareceram ao mesmo tempo), mas possuíram (por alguma razão) uma evolução diferente?

Exatamente. Sugiro consultar os livros:
O que é Evolução - Ernst Mayr - Rocco
O Livro de Ouro da Evolução - Carl Zimmer - Ediouro
Evolução - Mark Ridley - Artmed
Análise Evolutiva - Freeman & Herron - Artmed
Há muitos outros, mas esses são os melhores.

Professor, como ser um cosmologista?

Cosmologia é uma área de mestrado e de doutorado na pós-graduação em Física. É preciso, antes, fazer graduação em Física ou em Matemática, mas, neste caso, tem que fazer um nivelamento das matérias do Bacharelado em Física que não se vê no de Matemática, como Eletromagnetismo e Física Quântica, por exemplo. Há várias universidades que oferecem Cosmologia, bem como o CBPF, onde cursei, o Observatório Nacional e o Instituto de Física Teórica, todos do CNPQ.

Professor, dúvidas sobre a origem da vida são comuns e normalmente as hipóteses me agradam muito. Entretanto, há um impasse que não compreendo. Como a vida passou da animal para a vegetal ou vice-versa?

Não passou. Tanto os animais quanto os vegetais, as algas e os fungos se originaram de bactérias ou archeas, separadamente. Os animais passaram pelo estágio de protozoários.

wikipedia é confiável ? muitos professores dizem que não, mas nem tudo que esta la esta errado‎ bruno

A Wikipedia é uma valiosa ferramente de pesquisa, mas tem que ser usada com critério. Primeiro há que se ver se o artigo tem referências e consultá-las. Depois é bom consultar o mesmo assunto em mais de um idioma. A versão em inglês é bem melhor do que a em português. Os artigos costumam ser mais de vinte vezes mais extensos, mais bem escritos em termos de abrangência e profundidade e mais bem supridos de referências. Em espanhol também costuma ser melhor. Os artigos em português costumam ser melhores quando redigidos em Portugal.

Nossa galáxia tem nome próprio?‎

Sim. Via Láctea. Assim se chama porque ao visualizá-la a partir de nossa posição periférica a vemos como uma faixa leitosa no céu.

Qual a teoria que os cientistas mais aceitam sobre o fim do universo?‎

Na atualidade, face aos dados disponíveis sobre a densidade de massa-energia e a aceleração da expansão do Universo, o quadro mais considerado como verdadeiro é o do Big-Rip:http://pt.wikipedia.org/wiki/Big_Rip

• Estados Unidos • Irlanda • Finlândia • Japão • Coreia do Sul Se o senhor tivesse que escolher entre esses países para fazer intercâmbio na área de engenharia mecânica, qual desses escolheria?‎

Estados Unidos. Penso que lá é que são inventadas a maior parte das novidades mecânicas.

Professor, como o senhor aproveitaria sua vida se não precisasse mais trabalhar nem carecesse de dinheiro?‎

Iria me dedicar a escrever e proferir palestras, aproveitando para conhecer outros lugares. Para escrever, também estudaria muito. E fundaria uma ong ou uma fundação para abrigar minha biblioteca e dar cursos de atualização científica, filosófica, artística e cultural em geral, do mesmo modo que promoveria, nessa instituição, debates sobre temas polêmicos, como a existência de Deus, a origem do Universo, o Anarquismo, os problemas ecológicos e outros.

Se você fosse um deus, criaria raças inteligentes? Por quê?‎

Acho que, se houvesse Deus, ele seria um gozador que ficaria curtindo sua crianção como um brinquedo. Então seria bom criar não só uma, mas várias espécies inteligentes que coabitassem simultaneamente os planetas com vida, para ficar observando o que elas fariam umas com as outras. Se nos colocarmos como observadores extrínsecos à humanidade e contemplarmos a história em todos os lugares, realmente vamos achar muita graça de tanta tolice que tem sido feita. Parece que a humanidade adquiriu inteligência mas se recusa a fazer uso dela, de pirraça.

romantismo demais pode ficar brega?

Pode. Mas nem sempre ser brega é ruim. Pode ser que a pessoa que você goste aprecie. Então não tem problema. Pior é não ter romantismo nenhum por medo da breguice. O romantismo é uma característica das ações que confere mais relevância aos sentimentos, isto é, que valoriza gestos que expressem sentimentos. Isso é bom. Mas, particularmente, não acho que deva ser exagerado a ponto de se tornar melodramático. Há que se manter uma elegância e sobriedade, sem displicência, frieza e nem aspereza. Com educação e fidalguia, sem rebaixamento nem teatralidade exagerada. E, principalmente, com a mais completa sinceridade. Um romantismo fingido é pior do que a falta dele.

Acha possível a existência de mais de uma realidade?‎

Pode ser, mas eu entendo que não, uma vez que não há provas, evidências e nem indícios de nenhuma outra realidade, seja natural ou sobrenatural. O que existe são possibilidades de desenvolvimentos futuros do quadro global do Universo, que exibem uma imensa gama de possibilidades. Mas, uma vez que alguma delas se dê, as demais ficam perdidas e sempre um novo quadro de possibilidades vai se formando.

Ei Ernesto, você sabe por que o universo possui a necessidade de criar e destruir tudo?‎

Mas o Universo não possui essa necessidade. Tudo o que é destruído e criado o é por uma injunção de fatores que levam a tal desfecho sem necessidade nenhuma de assim o ser. É uma questão de coincidência de eventos que acarretam alguma destruição ou criação. Não há propósito e nem razão nenhuma para que as ocorrências do Universo sejam de tal ou qual modo.

Professor,qual seria o melhor curso a fazer na sua opinião...Economia ou Gestão de Empresas? E porquê?‎

O melhor curso a fazer é sempre aquele pelo qual se tenha mais afinidade e se compraza em exercer a atividade. Isso varia com a pessoa. Eu, por exemplo, preferiria Economia por ser mais teórico e acadêmico e menos prático. Mas há que goste mais de ser prático. Você é que tem que decidir em razão de seus pendores. Nunca se baseie, porém, na renumeração da atividade. O que importa é a satisfação de se fazer o que se gosta.

E se eu amar uma pessoa que mora longe de mim? Como posso saber se ela fala a verdade em relação aos seus sentimentos? Devo desistir de sermos felizes independente do tempo ou não?‎

Não há como saber de modo garantido. Ou se considera que ela diga a verdade ou não. Pode haver, é claro, manifestações comprobatórias do amor, por meio de ações concretas, como doações, por exemplo. Todavia, não se deve assumir compromissos, neste caso. Isso não impede de se amar e de se comprazer em se sentir amado. Desde que se saiba que pode não ser verdade e, com isso, não se desaponte ao ver que não seja. Mas pode-se curtir um amor, mesmo que não seja verdadeiro. Isso até pode ser gratificante. Não se deve desistir de ser feliz. Mas não se pode fechar à exclusividade desse amor não garantido.

Segundo as crenças,Deus não tem forma e, é na verdade um espirito?

Há muitas crenças a respeito de Deus. As religiões abrahãmicas (judaísmo, cristianismo e islamismo) consideram que Deus seja um espírito sem forma, sem volume, sem massa, sem contorno, sem superfície, sem cor, sem localização. Em suma, não se trata de um sistema físico. No entanto, seria capaz de agir sobre os sistemas físicos, alterando-lhes a posição, as dimensões, a composição, a estrutura, o movimento e tudo o mais. Como? As religiões não explicam.

Dá pra levar Astrologia a sério?‎

De modo nenhum. Não apenas porque não tem o menor fundamento, mas também porque não fornece respostas corretas.

Existe explicação para tudo no Universo?‎

Não. Inclusive há mais coisas que ainda não se conseguiu explicar do que já se conseguiu. E pode haver, até, fatos que não sejam nunca passíveis de explicação. Não há nenhuma obrigatoriedade de que tudo possua explicação. Apesar disso devemos sempre procurar a explicação. E se não a encontrarmos, aguardemos. No futuro pode ser que seja encontrada. Ou não. O que não se pode é inventar explicações quando não se as encontra. Como, por exemplo: "Deus que fez". Isso não explica nada, pois não se diz qual foi o procedimento de Deus para fazê-lo.

Os coalas tambem tem o 5 digito. Segundo a teoria da evolucao, porque eles nao evoluíram junto cono os macacos?‎

Os processos evolutivos, muitas vezes, conduzem a resultados coincidentes em ramos paralelos, por acaso. Não é necessário se estar em um ramo derivado em continuidade para se ter soluções análogas. Isso acontece, também, com o olho dos cefalópodes, por exemplo.

Gosto muito de Astronomia, porém na minha cidade não tem essa graduação. Faço Geografia, mas estou em dúvida se troco para Física, lembrando que gosto igualmente dos dois (mas meu foco para pós é a Astronomia). Qual curso entre Física e Geografia é o mais recomendando?‎

Para fazer Astronomia ou Astrofísica é bem melhor fazer Física, pois aquelas são ciências exatas derivadas da Física, além de envolver muita matemática que não é coberta no curso de Geografia. Você pode se dedicar à Geografia por diletantismo.

Professor, é possível sustentar um amor à distância ?‎

Certamente que sim. Nesse caso seria um amor "platônico", que não envolve contato físico, mesmo que o deseje, mas não seja possível. Trata-se de um amor que pode ser tão intenso, sincero e profundo quanto um amor fisicamente realizado. E pode despertar emoções e sentimentos do mesmo tipo, exceto as emoções do contato carnal, mesmo que não genital. É possível, inclusive, que esse amor seja uma paixão com dedicação exclusiva. E pode, também, perdurar pela vida toda dessa forma, mesmo que, concomitantemente, outro ou outros amores surjam. Isso não é tão bom quanto um amor que envolva o contado corporal, mas não quer dizer que seja indesejável. E melhor ter um amor platônico do que não ter amor nenhum. Ele pode ser muito significativo e dar sentido à própria vida. Não é preciso, pois, tentar deixar de amar a quem se ama platonicamente porque ele não vai poder se realizar em sua plenitude corporal. Inclusive porque se pode, também, amar a outrem tão verdadeiramente como a esse que é platônico e, com esse outro, se ter uma relação física. Desde que, é claro, todos os envolvidos saibam e estejam de acordo. Mesmo porque, não se consegue deixar de amar a ninguém porque se resolve fazê-lo.

Ernesto, como eu posso realmente saber se outra pessoa gosta de mim?‎

Por suas palavras, comportamento e atos. As palavras pode ser belas, mas mentirosas. Então há que sejam corroboradas por comportamentos e atos. Os comportamentos são os gestos, as expressões faciais e corporais e vários indícios da sinceridade das palavras. Os atos são o que se faz que, positivamente, confirmem o amor. Especialmente aquelas atitudes de atenção e cuidado, inclusive em prejuízo pessoal. Quem ama quer o bem da pessoa amada acima do próprio bem. Isso pode ser constatado. Ciúme, por exemplo, não é nenhuma prova de amor, mas de egoísmo. Uma coisa, porém, precisa ficar entendida. Amar, intensa, sincera e profundamente, não implica em amar exclusivamente. Pode haver amor múltiplo igualmente grande assim. O que o amor compreende é o cuidado, o carinho, a ternura, o desejo da companhia e do afeto, o prazer do contato corporal, não egoístico, isto é, que não se concentre na satisfação própria, mas que cuide de conceder o prazer também, o gosto do compartilhamento de interesses e atividades (mas não com possessividade exclusivista), o compromisso de dedicação, a construção de projetos comuns, o sonho de um futuro feliz. Se é amado(a) quando se sente feliz na companhia do amor, se se percebe que esse amor quer o nosso bem. Note que todo esse compromisso não necessariamente inclui a exclusividade e nem a perenidade. Como o disse Vinícius de Morais, "que seja infinito enquanto dure". Amor é algo maravilhoso que não pode ser conspurcado pelo egoísmo, pela possessividade, pela vigilância do ser amado, pelo cerceamento de sua liberdade. Todo amor tem que ser livre, como muito bem cantado por Marisa Monte na música "A Sua":


O que tu acha sobre a faculdade de Astrofísica? Qual é melhor, Astrofísica ou Astronomia?‎

Não conheço a existência de nenhum curso de graduação em Astrofísica. Só em Astronomia. Astrofísica é uma pós-graduação (mestrado e doutorado) da Física. Quanto à escolha, vai depender do que se gosta. Astronomia estuda a localização e movimento dos astros, Astrofísica sua composição e evolução. O ideal, para mim, é estudar ambas, além da Cosmologia que é o estudo da origem, estrutura e evolução do Universo como um todo.

O Sr desconsidera a hipótese da teoria da evolucao esta errada?‎

Sim, Até que seja sobejamente demostrado que o seja, a Evolução por Seleção Natural é a explicação correta para o surgimento das diversas espécies vivas. Não vejo outra alternativa. O criacionismo direto de cada espécie, é totalmente fantasioso. Não tem o menor cabimento, mesmo que se suponha a existência de algum Deus. E não há proposta nenhuma alternativa a essas duas.

Acha correto professor um namoro de 13 e 14 anos de idade?Qual a sua opniao sobre isso?‎

Não é correto nem incorreto. É aceitável. Isso depende da maturidade dos envolvidos. 13 anos, para mim, já contempla a maturidade para namorar. Mas acho que é preciso esperar ver se esse namoro seja, de fato, uma relação afetiva forte para passar a fazer sexo. O sexo já representa uma maior entrega mútua, de modo que a sua prática precoce pode ser prejudicial ao desenvolvimento psíquico. E fazer sexo de modo leviano não é bom também. Não porque a pessoa possa ficar rotulada de promíscua, mas porque vai fixar a ideia de que sexo seja uma leviandade, quando não é. É algo muito importante para o psiquismo. Não estou dizendo, puritanamente, que se tenha que manter a virgindade até o casamento, mas que a decisão de perdê-la tem que ser bem amadurecida.

Se Deus existe, o homem é escravo; mas, o homem pode e deve ser livre, pois Deus não existe. - Bakunin Concorda?‎

Inteiramente! O homem não pode nem deve ser livre. Ele "é" livre por natureza. Se sua liberdade for cerceada, ele tem que lutar para conquistá-la. Mas, às vezes, ele prefere viver escravo do que morrer pela liberdade. Isso já é uma escolha livre que ele faz.

professor, estou me interessando por várias áreas: Música, arte, filosofia, educação e biofísica. Estou com medo de não aproveitar tudo. O que o senhor me diz ?‎

Escolha uma das áreas para se dedicar profissionalmente, de preferência a mais difícil, e se dedique às demais por diletantismo. Assim faço eu. Escolhi a Física e a Matemática para o meu trabalho e me dedico à Filosofia, à Música, à Pintura, à Literatura e outros assuntos por diletantismo. Não é bom sufocar nada de que se interesse, pois ficará frustrado. Nem é preciso, como alguns dizem, se fixar em um assunto para ser bom nele e abandonar o resto. É perfeitamente possível ser bom em vários assuntos, mesmo díspares. Eu recomendaria você fazer curso em Biofísica, até o doutorado e ser professor disso, além de pesquisador, enquanto se dedicaria à Filosofia, à Música e às artes em geral por diletantismo.

Professor, é normal uma garota de apenas 17 anos não conseguir ter afeto por nenhuma outra pessoa além da família? Fora que nunca planejou e nem pensa em namorar/casar/ter filhos?!‎

Sim. Não há problema. Pode ser que o amor venha a surgir na vida dela ou não. Isso não é patológico. É uma orientação perfeitamente natural e legítima. Mesmo que a pessoa não sinta atração amorosa (quer erótica, quer apenas romântica), por outra pessoa (do sexo oposto ou do mesmo), ela pode sentir afeição não romântica e nem erótica por outras pessoas, isto é, amizade. E pode ser que isso preencha completamente suas carências afetivas. Mas pode ser que ela, também, não tenha esse tipo de carência, isto é, que baste a si mesma. Isso, também, não significa egoísmo ou egocentrismo. Pode ser um comportamento normal. Se isso não lhe traz nenhum desconforto, não é problema.

O universo pode ser considerado com um sistema fechado?

Claro que é. Uma vez que ele inclui tudo o que existe, não há "lado de fora" do Universo, isto é, nada fora dele para onde se poderia trocar energia, massa ou outra propriedade. Então ele é fechado e, "a fortiori", isolado. Todos os lugares existentes estão dentro do Universo. Não há conteúdo nenhum que possa sair ou entrar nele. Além do mais, mesmo se fosse finito, ele não teria fronteira, mas, nesse caso, seria fechado sobre si mesmo, como uma superfície esférica, bidimensionalmente considerada. Isso não significa que todas as propriedades sejam conservadas. Há as que são e as que não são. Densidade, temperatura, entropia, por exemplo não são conservadas. Massa-energia, carga elétrica, momento angular, momento linear o são. As três últimas têm um total nulo. A primeira ainda não se sabe o valor total, mas, pelo que se sabe, ele é conservado. Note que isso não é uma prescrição e sim uma descrição. As leis naturais não são morais. Elas não dizem o que deve acontecer, mas o que acontece. Se for verificada qualquer inobservância de alguma lei, ela tem que ser reformulada. Outra coisa é que toda lei natural só se aplica ao Universo existente. No surgimento do Universo, como não havia nada precedente, não havia lei a ser observada. Portanto não é impossível que o surgimento tenha se dado sem que massa-energia tivesse sido conservada. Mas pode ser que sim, se esse total for nulo, como pode ser que seja, Mesmo na consideração de que o surgimento do Universo tenha sido uma "criação" por parte de um agente extrínseco ao Universo (o que poderia ser isso?), esse agente teria provocado o surgimento de tudo sem que fosse proveniente de nada.

LinkWithin

Related Posts with Thumbnails